Tài liệu bồi dưỡng học sinh giỏi Toán Lớp 9

pdf 62 trang dichphong 5540
Bạn đang xem 20 trang mẫu của tài liệu "Tài liệu bồi dưỡng học sinh giỏi Toán Lớp 9", để tải tài liệu gốc về máy bạn click vào nút DOWNLOAD ở trên

Tài liệu đính kèm:

  • pdftai_lieu_boi_duong_hoc_sinh_gioi_toan_lop_9.pdf

Nội dung text: Tài liệu bồi dưỡng học sinh giỏi Toán Lớp 9

  1. Bài 1: Cho a,b là các số thực không âm. Chứng minh rằng : 23 2 3 1 1 a b b a 2 a 2 b . 4 4 2 2 1 Không mấy khó khăn ta có thể dự đoán được bất đẳng thức này trở thành đẳng thức khi a b . 2 1 1 a2 a,. b 2 b 4 4 Từ hai đánh giá này ta đưa bài toán về chứng minh bất đẳng thức mạnh hơn sau đây 1 1 1 1 a b b a 2 a 2 b , 2 2 2 2 Hay 2 1 1 1 a b 2 a 2 b . 2 2 2 Nhưng điều này hiển nhiên đúng vì theo bất đẳng thức AM-GM, ta có 2 1 1 2a 2 b 2 1 1 2 2 1 2a 2 b a b . 2 2 2 2 Bài toán được chứng minh xong. ■ Đây là một bất đẳng thức hai biến với hình thức không phức tạp nên ngoài chứng minh trên chúng ta có thể có thể chứng minh được bằng cách khai triển hai vế, lúc này cần lưu ý thêm về dấu đẳng thức để cân bằng hệ số thích hợp là được. Bài 2: Cho các số thực a,b,c>0. Chứng minh rằng: a()()() b c b c a c a b 3 2. bcb()()()2 c 2 cac 2 a 2 aba 2 b 2 Trước hết ta viết lại bất đẳng thức cần chứng minh như sau: a()()() b c b c a c a b 3. 2bc ( b2 c 2 ) 2 ca ( c 2 a 2 ) 2 ab ( a 2 b 2 ) Ta dự đoán rằng đẳng thức xảy ra khi a b c. Khi đó ta có b2 c 2 2 bc . Thế thì tại sao không thử dùng AM- GM nhỉ: 2bc b2 c 2 ( b c ) 2 2bc ( b2 c 2 ) . 2 2 a( b c ) 2 a Từ đó ta có . Kết hợp với hai đánh giá tương tự khác, ta suy ra 2bc ( b2 c 2 ) b c abc()()() bca cab a b c 2 . 2bcb (2 c 2 ) 2 cac ( 2 a 2 ) 2 aba ( 2 b 2 ) b c c a a b a b c 3 Cuối cùng, ta cần chỉ ra rằng , tuy nhiên đây lại là một đánh giá quen thuộc. b c c a a b 2 Bài toán kết thúc. Đẳng thức xảy ra khi và chỉ khi a b c . Bài 3: Cho các số dương x,y,z thỏa mãn x+y+z=1. Chứng minh rằng:
  2. x y z 1. x xyz y yzxz zxy Trước hết, sử dụng giả thiết và bất đẳng thức Cauchy - Schwarz, ta có xyz xxyz( ) yz ( xyxz )( ) x yz , do vậy x x . x x yz2 x yz Đến đây ta thiết lập thêm hai đánh giá tương tự để suy ra x y z x y z . x xyzy yzxz zxy 2 x yz 2 y zx 2 z xy Do vậy, để kết thúc chứng minh, ta cần chỉ ra rằng x y z 1. 2x yz 2 y zx 2 z xy Đặt a x, b y , c z, khi đó ta đưa bài toán về việc chứng minh a2 b 2 c 2 1. 2a2 bc 2 b 2 ca 2 c 2 ab Bất đẳng thức này tương đương với dãy sau 1a2 1 b 2 1 c 2 3 1, 2 2a2 bc 2 2 b 2 ca 2 2 c 2 ab 2 bc ca ab 1. 2a2 bc 2 b 2 ca 2 c 2 ab Đây là một đánh giá đúng vì theo bất đẳng thức Cauchy - Schwarz: bc ca ab bc2 2 ca 2 2 ab 2 2 2a2 bc 2 b 2 ca 2 c 2 ab 2 abcbc 222222222 2 bcaca 2 cabab ()ab bc ca 2 1, ab2 2 bc 2 2 ca 2 2 2 abcabc ( ) 1 do vậy ta kết thúc chứng minh ở đây. Đẳng thức xảy ra khi và chỉ khi x y z . 3 Bài 4: Cho các số thực dương a,b,c thoả mãn abc=1. Chứng minh rằng : 1 1 1 3 2(a b c ). a2 b 2 c 2 Do abc 1 và a,b,c là các số thực dương nên ta có thể dự đoán được dấu bằng xảy ra khi a=b=c=1. Dễ thấy rằng trong a,b,c tồn tại hai số cùng lớn hơn hoặc bằng 1, hoặc hai số đó cùng nhỏ hơn 1. Không mất tính tổng quát, ta có thể giả sử hai số đó là a,b, suy ra Có thể thấy rõ hơn điều này thông qua chú ý sau: [(a 1)( b 1) ][ ( b 1)( c 1) ][ ( c a )( a 1) ] ( a 1)2 ( b 1) 2 ( c 1) 2 0, do đó trong ba số (a−1)(b−1), (b−1)(c−1), (c−1)(a−1) có ít nhất một số không âm và do tính đối xứng nên ta hoàn toàn có thể giả sử (a−1)(b−1)≥0. (ab 1)( 1) 0 ab 1 ab 2( abc 1) 2( abc ).
  3. Ta sẽ chứng minh 1 1 1 1 1 1 3 2(ab c 1) 1 2( ab c ). a2 b 2 c 2 a 2 b 2 c 2 Đến đây, ta có thể sử dụng bất đẳng thức AM-GM cho hai số dương như sau: 1 2 2abc 1 1 2 2 abc 1 2ab , 2 c . c2 c c a 2 b 2 ab ab Cộng hai bất đẳng thức trên lại ta suy ra điều phải chứng minh. Đẳng thức xảy ra khi và chỉ khi a=b=c=1. ■ Bài 5: Chứng minh rằng với mọi số thực dương x,y,z thỏa mãn x(x+y+z)=3yz, ta có: (xy )3(3 xyyzzx )( )( )( zx )5( 3 yz ). 3 Do tính đồng bậc của bài toán ta giả sử x+y+z=3 từ đó ta có điều kiện đề bài tương đương x=yz. Vì x,y,z>0 nên áp dụng bất đẳng thức AM-GM ta có (y z )2 (3 x ) 2 (3 x ) 2 x yz x ( x 1)( x 9) 0. (1) 4 4 4 Ta lại có x+y+z=3 nên x<3<9.(2) Từ (1) và (2) suy ra 0<x≤1. Đến đây ta phân tích hai đại lượng đầu của bài toán ()()x y3 x z 3 cùng với việc sử dụng giả thiết x=yz: (xy )3 ( xz ) 3 (2 xyz ) 3 3( xyxzxyz )( )(2 ) ( x 3) 3 3(3 xyzx )( 3) ( x 3) 3 12 xx ( 3). Lại có ()()()(xyyzzx xyzxyyzzx )( ) xyz 3(3)[] x x x x2 124 x x 2 Và 5(y z )3 5(3 x ) 3 . Ta có bất đẳng thức cần chứng minh tương đương (x 3)3 12 x ( x 3) 3(12 x 4 x 2 ) 5(3 x ) 3 . Sau đó khai triển ta thu được (x 6)( x 3)( x 1) 0. Điều này đúng vì 0<x≤1. Bài toán được chứng minh xong. ■ Bài 6: Cho các số thực a,b,c thỏa mãn : a b 1 a 3 . Chứng minh : a + b − c ≤ 4. ab 6 ab 6 c Khi giải toán bất đẳng thức, lúc nào ta cũng nghĩ đến việc làm sao để sử dụng giả thiết cho hiệu quả, bài này cũng không ngoại lệ. Quan sát giả thiết của đề bài, các bạn có thấy chỉ có một giả thiết liên quan đến biến c thôi không? Như vậy, để có thể chứng minh a b c 4 thì chắc chắn ta phải sử dụng giả thiết này vào rồi. Lúc này, ta đưa được bài toán về chứng minh ab a b 4. 6
  4. Bây giờ, xem xét tiếp, ta thấy rằng giả thiết cho ta 3≥a≥b≥1, kết hợp với dự đoán dấu bằng sẽ xảy ra khi a=3, đồng thời ta cần có sự xuất hiện của a+b và ab trong các đánh giá của mình (do bất đẳng thức cần chứng minh nó như vậy mà), ta nghĩ đến đánh giá sau đây (3 a )(3 b ) 0. Từ đánh giá này, ta suy ra 3(a+b)≤ab+9, hay ab a b 3 . 3 (Ta thực hiện đánh giá a+b theo ab vì giả thiết bài toán thì có liên quan đến ab nhưng không có chút dính dáng nào tới a+b cả). ab ab Vậy là, ta chỉ cần chứng minh 3 4,hay ab 6. 3 6 Cái này đúng do giả thiết nè. Và như vậy là với một số phân tích đơn giản, ta giải xong bài toán rồi đó. Vui hé. ■ Bài 7: Cho a,b,c là các số thực dương. Chứng minh bất đẳng thức: abcabbcca2 2 2 2 2 2 9 5( abc ). Đặt x=a−1,y=b−1,z=c−1, khi đó bất đẳng thức ban đầu được viết lại thành (x 1)2 ( y 1) 2 ( z 1) 2 ( xy 1)( 1) 2 ( yz 1)( 1) 2 ( zx 1)( 1) 2 9 5( xyz 3), hay (sau khi đã khai triển và rút gọn) 2(x2 y 2 z 2 ) 2( xy yz zx ) xy 2 yz 2 zx 2 0. Lưu ý rằng từ phép đặt trên, ta suy ra x,y,z>−1, do đó đánh giá cuối cùng đúng vì 2(x2 y 2 z 2 ) 2( xyyzzxxy ) 2 yz 2 zx 2 ( xyz ) 2 yx 2 ( 1) zy 2 ( 1) xz 2 ( 1) 0. Và như vậy bài toán được chứng minh xong. Đẳng thức xảy ra khi và chỉ khi a=b=c=1. ■ Bài 8: Cho x,y,z>0 thoả mãn x+y+z=1. Tìm giá trị nhỏ nhất của biểu thức : x2()()() y z y 2 z x z 2 x y P . yz zx xy Trước hết ta viết biểu thức bài cho lại như sau x2 y 2 z 2 x 2 y 2 z 2 P . y z x z x y Tiếp đến ta sẽ chứng minh x2 y 2 z 2 x y z. (1) y z x x 2 Sử dụng AM-GM cho hai số y 2 x , y
  5. rồi thực hiện tương tự chi hai đại lượng còn lại sau đó cộng vế lại với nhau ta sẽ được bất đẳng thức (1). x2 y 2 z 2 Hoàn toàn tương tự cũng có x y z. (2) z x y Cộng vế theo vế của (1) và (2), lại ta được P 2( x y z ) 2. 1 Kết luận: Vậy giá trị nhỏ nhất cần tìm là P=2 đạt được khi và chỉ khi x y z . 3 Cách 2 Áp dụng bất đẳng thức Cauchy-Schwarz, ta có xyzyzxzxy2()()()() 2 2 xyz 2 P . yz zx xy yz zx xy y z z x x y yz y z Theo một hệ quả trực tiếp của bất đẳng thức AM-GM, ta lại có (y z )2 4 yz ,nên . y z 4 Ta suy ra thêm hai đánh giá tương tự khác để có ()x y z 2 P 2( x y z ), y z z x x y 4 4 4 và vì x+y+z=1 nên ta suy ra P 2. 1 Cuối cùng, với x y z (thoả mãn điều kiện) thì P=2 nên ta kết luận 2 là giá trị nhỏ nhất của biểu thức P. Bài 3 toán kết thúc. ■ Cách 3 Sử dụng bất đẳng thức AM-GM, ta có x2( y z )2x2 yz 2 x 2 4 x 2 . yz yzyz y z Thực hiện tượng tực cho hại đại lượng còn lại sau đó cộng tương ứng các vế lại với nhau ta thu được x2 y 2 z 2 P 4 . y z z x x y Mặt khác theo bất đẳng thức Cauchy-Schwarz thì x2 y 2 z 21 ( x y z ) 2 x y z 1 ·. y z z x x y2 x y z 2 2
  6. 1 Từ đó suy ra P 2. Mặt khác dễ thấy với x y z thì P=2. Việc tìm được các giá trị của x,y,z thỏa mãn điều 3 kiện bài toán và P=2 cho phép ta kết luận giá trị nhỏ nhất của P là 2. Bài toán được giải quyết xong. □ Bài 9: Cho a,b,c là các số thực dương thỏa mãn điều kiện ab+bc+ca=a+b+c. Chứng minh rằng : a b b c c a 3. a2 b 2 b 2 c 2 c 2 a 2 ()a b 2 Đánh giá quen thuộc a2 b 2 Ta có 2 a b b c c a 2 2 2 . abbcca2 2 2 2 2 2 abbcca Bất đẳng thức tương đương 2 2 2 3(a b c ) . ab bc ca abbcca ab bc ca ab Ta nhân ab+bc+ca lên hai vế và để ý c , từ đó viết được bất đẳng thức dưới dạng a b a b 2ab 2 bc 2 ca a b c. a b b c c a xy x y Tới đây thì ngon rồi, ta nghĩ ngay đến việc sử dụng đánh giá quen thuộc và thu được ngay kết quả. x y 4 Tuyệt. ■ Bài 10: Cho các số thực a,b,c∈[0,1] thỏa mãn điều kiện a+b+c=2. Tìm giá trị lớn nhất của biểu thức sau : P a2 4 a 5 b 2 4 b 5 c 2 4 c 5. Thứ nhất, dự đoán dấu bằng để đạt giá trị lớn nhất. Không khó để ta có thể dự đoán một cách "tự nhiên" là khi a=b=1,c=0 thì biểu thức đạt giá trị lớn nhất. Rõ ràng đây là bộ vừa thỏa mãn điều kiện đề bài mà nó trông cũng rất "tròn trịa" mà thường cái gì "tròn trịa" đều đạt hiệu quả cao. Tuy vậy, việc dự đoán không phải lúc nào cũng đúng. Nhưng trong trường hợp này, niềm tin của ta được củng cố bởi sự "tròn trịa". Như vậy, ta cũng nên bỏ chút thời gian để đi theo cái dự đoán đó, nếu không hiệu quả, ta đi theo hướng khác, nó sẽ giúp cho ta tiếp thu thêm kinh nghiệm làm bài. Thứ hai, "liều" với dự đoán ở trên, ta tìm cách đánh giá cho hiệu quả. Quan sát thấy biểu thức của ta là tổng đối xứng ba biến và hơn nữa nó có thể viết lại dưới dạng P=f(a)+f(b)+f(c),
  7. với f( x ) x2 4 x 5 Như vậy, một cách tự nhiên, ta mong một đánh giá "riêng lẻ" hay "chia để trị" kiểu f( x ) x2 4 x 5 mx n , đúng với x [0,1]với m,n là các số thực nào đó. Để khi đó, ta có thể có Pfa ()()() fb fcmabc ( )323. n n m Nói một cách khác, ta đã tìm được một chặn trên của P mà ta mong đó là giá trị lớn nhất. Công việc của ta là đi tìm m,n. Như đã nói ở trên, ta cần đánh giá để đảm bảo dấu bằng, mà vì các biến là đối xứng với nhau nên ta không thể quyết định biến nào bằng 0 hay biến nào bằng 1 khi biểu thức P đạt giá trị lớn nhất. Bởi vậy, tốt nhất, ta sẽ chọn m,n sao cho mà dấu bằng tại 0 và 1 đều thỏa. Nói một cách khác, ta cần tìm m,n sao cho phương trình x2 4 x 5 ( mx n ) 0, có hai nghiệm là 0 và 1. Thay x=0 và x=1, ta tìm được m 2 5 và n 5. Như vậy, ta mong là bất đẳng thức sau đúng x2 4 x 5 2 5 x 5. Nhưng kì diệu thay, bất đẳng thức này hoàn toàn đúng với điều kiên x [0,1] Như vậy, ta sẽ có P f() a f () b f () c 2 5( a b c )3522 5. Với a=b=1,c=0 ta có ngay P 2 2 5. Bài 11: Với a,b,c là các số thực dương thỏa mãn điều kiện a+b+c=abc. Chứng minh bất đẳng thức sau đây : a b c 1. b3 c 3 a 3 Bất đẳng thức này không đồng bậc, nhận thấy vế trái bậc (−2), vế phải bậc 0, điều kiện giả thiết a+b+c (bậc 1) =abc (bậc 3), từ đó ta có ý tưởng làm cho hai biểu thức đồng bậc. Với ý tưởng như vậy ta sẽ tiến hành như sau. Bất đẳng thức cần chứng minh tương đương với a b c abc 3 3 3 a b c, b c a Hay a2 c b 2 a c 2 b a b c. b2 c 2 a 2 Đến đây sử dụng bất đẳng thức Cauchy-Schwarz, ta có 2 1 1 1 a2 c b 2 a c 2 b a b c 2 2 2 . c a b b c a b c a Ta cần chứng minh
  8. 2 a b c 1 1 1 (),a b c b c a a b c Hay a2 b 2 c 2 a b c a b c 3 . b2 c 2 a 2 c a b b c a Theo bất đẳng thức AM-GM, ta có a2 b 2 c 2 a b c 2 1 2 1 2 1 2 b c a b c a Và a b c a c b 6. b c a c b a Cộng lại, ta có điều phải chứng minh. Đẳng thức xảy ra khi và chỉ khi a b c 3. Cách 2 Ta viết bất đẳng thức cần chứng minh lại dưới dạng thuần nhất như sau a b c a b c . b3 c 3 a 3 abc Nhân hai vế của bất đẳng thức này với abc>0, ta được ab2 bc 2 ca 2 a b c. c2 a 2 b 2 Sử dụng bất đẳng thức AM-GM, ta có bc2 ca 2 bc 2 ca 2 b 33 . . b 3 c . a2 b 2 a 2 b 2 Thực hiện đánh giá tưong tự cho các đại lượng còn lại sau đó cộng tương ứng các vế lại với nhau ta thu được kết quả cần phải chứng minh. □ Bài 13: Cho các số thực a,b>0 thỏa mãn 2(a2 b 2 ) ab ( a b )( ab 2) . Tìm giá trị nhỏ nhất của biểu thức: a3 b 3 a 2 b 2 P 4 3 3 9 2 2 b a b a a b Đặt ẩn phụ t , quy được P về dạng hàm một biến theo t, rồi sau đó chặn miền cho t bằng cách sử dụng bất b a đẳng thức a b 2 2 2 2 1 (a b ) 1 ( a b ). . b a ab ab
  9. Cuối cùng, ta đưa được bài toán về khảo sát hàm môt biến trên miền vừa chặn được. Bài 14: Chứng minh rằng với mọi số thực dương a,b,c, ta luôn có : 2 2 2 ab bc ca2 2 2 3 3 3 a b c a b c 2 ( ab bc ca ) . c a b b c a Với bất đẳng thức chứa căn, điều đầu tiên ta phải làm đó là tìm cách phá căn. Trong bài toán này, để làm được điều đó, ta liên tưởng ngay tới bất đẳng thức AM-GM dạng 2 xy x y Nguyên tắc tiếp theo, khi sử dụng AM-GM, cần phải đảm bảo điều kiện đồng bậc của x và y, đó là lí do ta không nên đánh giá như sau: 3 3 3 a b c 3 3 3 a b c 2 (ab bc ca ) ab bc ca b c a b c a ab2 ab 3 Mặt khác, để ý rằng Từ đó, có lời giải sau. c bc Đây là một bất đẳng thức hoán vị nên không mất tính tổng quát, ta hoàn toàn có thể giả sử a là số nằm giữa a,b,c. Sử dụng bất đẳng thức AM-GM dạng 2 xy x y 3 3 3 3 3 3 abc abbcca abc 2 (ab bc ca ) bc b c a bc b c a Vậy chứng minh sẽ hoàn tất nếu ta chỉ ra được rằng 3 3 3 2 2 2 ab bc ca a b c ab bc ca 2 2 2 bc a b c bc b c a c a b Hay ab2 a 3 bcabbcca 2 2 2 2 c2 ca b 2 a 2 b 2 c 2 c b a c a b Tương đương với a( a b )( a c ) 0 b Hiển nhiên đúng do giả sử a là số nằm giữa a,b,c. Chứng minh hoàn tất. Dấu bằng xảy ra khi và chỉ khi a=b=c>0 1 2 3 Bài 15: Cho x,y,z∈[0,1] và thỏa mãn 1 4x 5 4 y 5 4 z 5
  10. Tìm giá trị lớn nhất của : P xy2 z 3. Bài này ta dùng bất đẳng thức AM−GM. Cụ thể: Viết giải thiết thành: 1 1 1 1 1 1 1 454545454545x y y z z z Ta có 11111111111 4x 5545545545545545 y y z z z 1 4y 1 4 y 1 4 z 1 4 z 1 4 z 545545545 y y z 545 z 545 z Áp dụng bất đẳng thức AM−GM,ta có: 1 y2 z 3 45 (1) 4x 5 (4 y 5)2 (4 z 5) 3 Tương tự 1 xyz3 45 (2) 4y 5 (4 x 5)(4 y 5)(4 z 5)3 1 xyz3 45 (3) 4z 5 (4 x 5)(4 y 5)(42 z 5) 2 Bình phương hai vế của bất đẳng thức (2) và lập phương hai vế của bất đẳng thức (3) rồi nhân vế theo vế của (1),(2),(3) , ta được: 1 46xy 2 z 3 Do đó 1 xy2 z 3 46 1 Đẳng thức xảy ra khi và chỉ khi: x y z 4 1 Vậy giá trị lớn nhất của P bằng 46 Bài 16: Cho 4 số thực a, b, c, d thỏa mãn: abcd a2 b 2 c 2 d 2 .Chứng minh rằng: abcd a b c d 8 Áp dụng bất đẳng thức AM-GM ta có abcd a2 b 2 c 2 d 2 42 abcd abcd 16
  11. Áp dụng bất đẳng thức Cauchy-Schwarz ta có ()a b c d 2 abcd a2 b 2 c 2 d 2 . 4 Đặt k=a+b+c+d nên ta cần chứng minh k2 4 k 320 ( k 8)( k 4)0 Nếu a+b+c+d 0. Trở lại bài toán cần xét, khi đọc tới giả thiết x2 y 2 z 2 3 thì phải hiểu rằng Max P=3k và công việc của ta là tìm k để (*) luôn đúng với mọi x,y,z. Để ý rằng, (*) có thể viết lại thành ky2 y( x z ) kx 2 kz 2 2 xz 0 Do hệ số k>0 nên coi đây là tam thức bậc hai ẩn y thì dễ thấy nếu ta tìm được k sao cho Δ≤0∀x,z, bài toán sẽ được giải quyết Ta lại có: (1 4k2 )( x 2 z 2 ) 2(1 4 k ) xz (1) Mặt khác, bất đẳng thức trên đối xứng với x và z nên ta có thể dự đoán P đạt Max khi và chỉ khi x=z. Từ đó, dẫn tới ý tưởng thay x=z=1 vào (1), ta thu được 2k2 2 k 1 0 1 3 Do k>0 nên ta chỉ chọn nghiệm k 2 3(1 3) Vậy kết luận được Max P . 2 Bài 18: Tìm giá trị lớn nhất và giá trị nhỏ nhất của hàm số : y 12 4 x x2 x 2 2 x 3 Tập xác định D  1;3 . Dễ thấy rằng trên (−1;3) thì 2 x 1 x y 12 4x x2 x 2 2 x 3 nên y′=0 tưong đương với (2) x x2 23(1) x x x 2 412 x
  12. x 0 2 2 2 2 (2x )( x 2 x 3)(1 x )( x 4 x 12) 5 x 4 Thử lại ta thấy chỉ có x=0thỏa mãn phương trình y′=0. Tiếp đến ta lại có f(1) 7, f (0) 3; f (3) 15. So sánh các giá trị trên suy ra miny 3,max y 15. Bài 19: Cho các số thực không âm x,y thỏa mãn x+y=1.Tìm giá trị lớn nhất và nhỏ nhất của biểu thức: P 1 x2011 1 y 2011 Thay y=1−x và biểu thức đã cho, ta có: P 1 x2011 1 (1 x ) 2011 f ( y ) Ta có 2011x2010 2011 (1 x ) 2010 f ( y ) . . f ( x ) f (1 x ) 21 x2010 2 1 (1 x ) 2011 t2010 Với f() t là một hàm đồng biến. 1 t2011 Đến đây so sánh x với 1−xvà lập bảng biến thiên nữa là xong. ■ Bài 20: Cho x,y,z>0 thỏa mãn điều kiện xyz=8. Tìm giá trị lớn nhất của biểu thức: 1 1 1 P . 2x y 6 2 y z 6 2 z x 6 x y z Đặt a2,, b 2 c 2 với a,b,c>0 thì ta cũng có abc=1. Biểu thức trở thành: 2 2 2 1 1 1 2P . 2a2 b 2 3 2 b 2 c 2 3 2 c 2 a 2 3 Áp dụng bất đẳng thức AM-GM, ta có: 1 1 1 1 1 2PP . 2222222222a ab b bc c ca 4 Bài 21: Cho các số thực khác không x,y thỏa mãn xy(). x y x2 y 2 xy . Tìm giá trị lớn nhất của biểu thức sau : 1 1 P . x3 y 3 Ta có
  13. 1 1 1 1 xy( x y ) x2 y 2 xy 1 x y x2 y 2 1 1 Đặt a ; b Điều kiện bài toán trở thành: a b a2 b 2 ab x y Bài toán trở thành P a3 b 3 Ta có Pab 3 3()3()()( ab 3 abab aba 2 bab 2 )() ab 2 Ta có 3(a b )2 ( a b ) 2 a b ( a b )2 3 ab ( a b ) 2 4 4 4(a b )( a b )2 ( a b )( a b 4)00 a b 4 1 Suy ra P 16 Dấu bằng xảy ra khi: a=b hay x y 2 Bài 22: Chứng minh rằng với mọi số thực x,y,z, ta có: x2 xyy 2 y 2 yzz 2 x 2 xzz 2 . x3 z 3 x z 3 3 Xét số phức z y xi,, z y zi Ta có: z z () x z i 12 2 2 2 2 1 2 2 2 2 2 Áp dụng bất đẳng thức|z1 | | z 2 | | z 1 z 2 |ta có: x2 xy y 2 y 2 yz z 2 x 2 xz z 2 . nên ta suy ra điều phải chứng minh. Bài toán kết thúc.■ Bài 23: Cho các số thực a,b,c đôi một khác nhau thuộc đoạn [0;2] 1 1 1 Tìm giá trị nhỏ nhất của biểu thức: P ()()()a b2 b c 2 c a 2 1 1 8 Áp dụng BĐT ta có x2 y 2() x y 2 8 1 9 9 P (c a )2 ( c a ) 2 ( c a ) 2 4 Có "=" chẳng hạn khi a=0,b=1,c=2. a2 b 2 c 2 Bài 24: Cho a,b,c là các số thực dương. Chứng minh rằng: 1 2a2 bc 2 b 2 ac 2 c 2 ab Để ý rằng, theo bất đẳng thức AM-GM dạng 2xy x2 y 2 ta có
  14. 2a2 1 1 1 a 2 b 2 c 2 a 2 2bc 1 1 2 2 2 2 2 2 2a bc1 1 1 a b b c c a 2a2 a a a2 a 2 2. . . b c b2 c 2 Thiết lập hai bất đẳng thức tương tự và cộng lại, ta thu được 2a2 2 b 2 2 c 2 2( a 2 b 2 b 2 c 2 c 2 a 2 ) 2 2a2 bc 2 b 2 ca 2 c 2 ab ab 2 2 bc 2 2 ca 2 2 Hay a2 b 2 c 2 1 2a2 bc 2 b 2 ca 2 c 2 ab Cách 2 1 1 1 Bất đăng thức đã cho tương đương với: 1 bc ca ab 2 2 2 a2 b 2 c 2 bc ca ab Đặt x2 ,, y 2 z 2 với x,y,z>0, ta có: xyz=1. a2 b 2 c 2 Bất đẳng thức cần chứng minh trở thành: x2 y 2 z 2 1 2 x2 2 y 2 2 z 2 Mặt khác, theo bất đẳng thức Cauchy−Schwarz, ta có: x2 y 2 z 2() x y z 2 2 x2 2 y 2 2 z 2 6 x 2 y 2 z 2 Ta cần chỉ ra ()x y z 2 1 6 x2 y 2 z 2 nữa là bài toán được giải quyết. Thật vậy, bất đẳng thức trên tương đương với: xy yz zx 3 Bất đẳng cuối luôn đúng theo bất đẳng thức AM−GM. Từ đó, ta có điều phải chứng minh ■ Bài 25: Cho các số thực x,y thỏa mãn bất phương trình 5x2 5 y 2 5 x 15 y 8 0 . Tìm giá trị nhỏ nhất của biểu thức S x 3y. Do S=x+3y nên x=S−3y, thay vào giả thiết 5x2 5 y 2 5 x 15 y 8 0 và viết theo hệ số của biến y ta thu được 50y2 30 Sy 5 S 2 5 S 8 0
  15. Vì bất đẳng thức trên đúng với mọi y nên ta có Δ≥0, tức là 900SSS2 4.50.(5 2 5 8) 0 2 Biến đổi tương đương ta thu được 100SSS 1000 1600 0 2 8 . Bài 26: Cho a,b là các số thực thỏa mãn a2 b 2 4 a 3 b . Tìm giá trị lớn nhất và nhỏ nhất của biểu thức P 2a 3b. Ta có 2 2 2 2 3 25 a b 4 a 3 b ( a 2) b . 2 4 3 2 2 25 1 3 Đặt x a 2, y b , x y và P 2( a 2) 3 b 2 x 3 y 2 4 2 2 Áp dụng bất đẳng thức Cauchy-Schwarz ta có 1 13.25 (P )(23)13(2 x y 2 x 2 y 2 ) . 2 4 5 1 5 1 5 13 1 5 13 Do đó ta có 13 P 13, hay P 2 2 2 2 2 Cách 2 P 2 a Ta có P 2 a 3 b b 3 Thay vào biểu thức phía trên ta được: P 2 a P 2 a a2 ( )43( 2 a )13 a 2 2(272)9 P a P P 2 0 3 3 Ta cần tìm P để phương trình trên tồn tại a. Tức là ta phải có: 94513 94513 1513 1513 i 9PPPP2 9 729 0 18 18 2 2 Cách 3 *Trường hợp 1: a=0 và b=0 thì P=0 *Trường hợp 2: a≠0 và b≠0 thì : (2a 3)(4 b a 3)8 b a2 6 ab 9 b 2 P 2 a 3 b a2 b 2 a 2 b 2 Đặt: a=t.b ( t≠0) dẫn đến: 8t2 6 t 9 P t2 1
  16. 8t2 6 t 9 Đến đây chúng ta chỉ cần xét hàm: f() t với t≠0 là bài toán được giải quyết. t2 1 Bài 27: Cho các số dương a,b,c thỏa mãn abc+a+c=b . Tìm giá trị lớn nhất của biểu thức 2 2 3 P . a2 1 b 2 1 c 2 1 b a Từ giả thiết, ta có b a và c . suy ra 1 ab 1 1 (1 ab )2 ( b a ) 2 1 . 1 c2()b a 2 (1 a 2 )(1 b 2 ) (1 a 2 )(1 b 2 ) 1 (1 ab )2 Như vậy, ta có thể viết lại P dưới dạng 2 2 3(b a )2 ( b a )(5 a b ) P 3 3. 1 a2 1 b 2 (1 a 2 )(1 b 2 ) (1 a 2 )(1 b 2 ) Bây giờ, sử dụng các bất đẳng thức AM-GM và Cauchy-Schwarz, ta có 2 1 1(33)(5 b a a b ) ( a b )(2 a 2 1)(1 b 2 ) b a)(5)·(33)·(5) a b b a a b . 3 3 2 3 3 1 10 Do đó P 3 . 3 3 Bài 28: Cho các số x,y,z thỏa mãn 1 x , y , z 4 , x y, x z . Tìm giá trị lớn nhất và giá trị nhỏ nhất của biểu thức x y z P 2x 3 y y z x z Đặt x y z f(,,) x y z 2x 3 y y z x z Ta có x 2 y f(,,) x y xy 2x 3 y x y Xét hiệu ()()xy z2 x y f( x , y , z ) f ( x , y , xy ) 0 Hiển nhiên đúng. (x z )( y z )( x y ) Ta có
  17. x x 2 y y 2 P f(,,) x y z 2x 3 y x yx x 2 3 1 y y x Ta đặt t ,1 t 2 ta có y t2 2 P 2t2 3 1 t Cách 2 1 1 1 Ta có bổ đề sau: với ab≥1 1 a2 1 b 2 1 ab Thật vậy, trừ vế theo vế, ta có ngay: (ab 1) (a b )2 0 Hiển nhiên đúng. (1 a2 )(1 b 2 )(1 ab ) x Vì 1khi đó y x x x y z y1 1 y 2 P 2x 3 y y z x z x z x x x 2 3 1 1 2 3 1 y y z y y Cách 3 y z x Đặt a;;. b c x y z Khi đó abc=1 và 2 bc 1. ta có 1 1 1 P . 2 3a 1 b 1 c 1 Xét bài toán mới này có các biến b và c bình đẳng nên ta dự đoán đẳng thức xảy ra khi b c . a 1 2 a 1 Khi đó P :() f a với a ;1 . 2 3a 1 a 4 Bài 29: Cho các số x,y,z thỏa mãn1 x , y , z 9 và x y, x z . Tìm giá trị lớn nhất và giá trị nhỏ nhất của biểu x y z thức P x 2 y y z x z
  18. x y z Đặt f(,,) x y z x 2 y y z z x Suy ra y z 2y ( x y )( xy z )2 f( x , y , z ) f ( x , y , xy ) 0 y z z x x y( y z )( z x )( x y ) x Vì x=max{x,y,,z} nên ta đặt t với 1 t 3 y t2 2 Xét hàm f() t trên 1;3 t2 2 t 1 Ta có 2(t3 2)( t 2) f ( t ) , f ( t ) 0 t 2  t 3 2 (t2 2) 2 ( t 1) 2 Dễ thấy giá trị t 3 2 không thỏa đk 7 4 Từ đó ta dễ dàng kết luận được: MinP ; MaxP 6 3 x2 y y 2 z z 2 x Bài 30: Cho : x≥y≥z>0 . Chứng minh rằng: x2 y 2 z 2 z x y y z x z x y Áp dụng BĐT Bunhiacopsky cho 2 dãy số x;; y z và x;; y z z x y y z x xy2 yz 2 zxxz 2 2 yx 2 zy 2 Ta có )( ) (x2 y 2 z 2 ) 2 (1) z x y y z x Xét hiệu xy2 yz 2 zx 2 xz 2 yx 2 zy 2 F () z x y y z x 1 ()xz32 yz 32 zx 32 xz 32 yz 32 zy 32 xyz 1 [(xz32 yz 32 ) ( yz 32 zy 32 ) ( zx 32 xz 32 )] xyz 1 (x yy )( zx )( zxy )( yz zx ) 0 xyz Suy ra xy2 yz 2 zx 2 xz 2 yx 2 zy 2 (2) z x y y z x
  19. Từ (1) và (2) ta được xy2 yz 2 zx 2 xy 2 yz 2 zxxz 2 2 yx 2 zy 2 ( )2 ( )( ) (x 2 y 2 z 2 ) 2 z x y z x y y z x Vậy xyyzzx2 2 2 xyyzzx 2 2 2 ()() 2 x 2 y 2 z 2 2 x 2 y 2 z 2 z x y z x y Đẳng thức xảy ra ⇔x=y=z>0■ a2 bc b 2 ca c 2 ab Bài 31: Chứng minh rằng với mọi a,b,c > 0ta có: a b c b c c a a b Ta có a2 bc a 2 abbcca ( abac )( ) a a . b c b c b c Tương tự ta có bất đẳng thức ban đầu tương đương với: (abac )( )( bcba )( )( cacb )( ) 2(a b c ) b c a c a b Đặt a+b=x;b+c=y;c+a=z ta có xy yz zx x y z z x y Bài 32: Cho a,b,c là các số thực dương thỏa mãn a+b+c=1. Chứng minh rằng 1 1 1 a2 b 2 c 2 82. a2 b 2 c 2 1 Ta dễ dàng dự đoán rằng đẳng thức xảy ra tại a b c ,khi đó bằng cách nhân cả hai vế cho 82 và áp dụng bất 3 đẳng thức Cauchy-Schwarz, ta có: 2 1 9 (1 81) a 2 a a a Áp dụng bất đẳng thức AM-GM, ta được 9 81a 80 a 54 80 a a Đánh giá tương tự cho b,c và cộng lại ta có điều phải chứng minh .□ x4 y y 4 z z 4 x 3 Bài 33: Cho các số dương x,y,z>0,xyz=1. Chứng minh rằng: x2 1 y 2 1 z 2 1 2
  20. x4 y Thoạt nhìn biểu thức có lẽ ai cũng muốn đánh giá x2 1 2 x tuy nhiên như vậy bất đẳng thức sẽ bị đảo x 2 1 chiều. Chính điều này gợi cho ta ý tưởng sử dụng kĩ thuật AM-GM ngược dấu. Hãy nhớ nguyên tắc: Phủ định của phủ định là khẳng định. x4 y x 2 y x 2 y xy Thật vậy, để ý rằng, x2 y x 2 y x 2 y x2 1 x 2 1 2 x 2 Thiết lập hai bất đẳng thức tương tự và cộng lại, ta thu được xy4 yz 4 zx 4 xyyzzx x2 y y 2 z z 2 x x2 1 y 2 1 z 2 1 2 Từ đó, ta sẽ thử chứng minh bất đẳng thức mạnh hơn là xy yz zx 3 x2 y y 2 z z 2 x hay 2(xy2 yz 2 zx 2 ) xy yz zx 3 2 2 Sử dụng bất đẳng thức AM-GM bộ ba số dạng a b c 33 abc dễ thấy x2 y y 2 z z 2 x 3.3 x 2 y . y 2 z . z 2 x 3 xyz 3 Vậy chứng minh sẽ hoàn tất nếu ta chỉ ra được xy2 yz 2 zx 2 xy yz zx Một lần nữa, sử dụng bất đẳng thức AM-GM bộ ba số kết hợp với giả thiết xyz=1, ta có x2 y x 2 y y 2 z 3.3 x 2 y . x 2 y . y 2 z 3. 3 x 4 y 4 z 3 xy Thiết lập hai bất đẳng thức tương tự và cộng lại theo vế, ta thu được 3(xy2 yz 2 zx 2 ) 3( xy yz zx ) hay xy2 yz 2 zx 2 xy yz zx Từ đó suy ra điều phải chứng minh. Dấu đẳng thức xảy ra khi a=b=c=1. 34 Có logba.logcb.logac=1 Để có được điều này ta chỉ cần sử dụng một bất đẳng thức quen thuộc đó là AM−GM ta có : logba log c b log a c 1 33 ab bc ca ( abbcca )( )( ) Mặt khác cũng theo bất đẳng thức AM−GM ta lại có: (ab )( bc )( ca )2( abc )3(3 abbcca )( )( ) Hay 1 3 3 (a b )( b c )( c a ) 2( a b c )
  21. Do đó loga log b log c 9 b c a a b b c c a2( a b c ) Dấu "=" xảy ra khi và chỉ khi a=b=c>1 Bài 35: Cho các số thực không âm x,y,z thỏa mãn: x+y+z=1. Tìm giá trị lớn nhất của: P 9xy 10yz 11zx Để ý rằng, với giả thiết x+y+z=1 thì P 9xy 10yz 11zx 9xy z 10y 11x 9xy 1 x y 10y 11x Khai triển và rút gọn, ta thu được P 11 x2 10 y 2 11 x 10 y 12 xy Tương đương với 11x2 (12 y 11) x 10 y 2 10 y P 0 * Coi đây là tam thức bậc hai ẩn x, do điều kiện tồn tại của x nên suy ra phải có nghiệm, tức (12y 11)2 44(10 y 2 10 y P ) 0 Hay 296y2 176 y 121 44 P 0 Tương đương 74 2 22 121 P y y 11 37 296 Dùng phép tách thành bình phương, dễ dàng nhận thấy 22 121 5445 y2 y 37 296 10952 74 5445 495 Suy ra P . 11 10952 148 495 Vậy kết luận P 148 2 2 2 Bài 36: Cho a,b,c>0 thỏa mãn: a+b+c=3 . Chứng minh rằng: (a c )( b 1) abc ( a b c 1) Nhìn vào bài này ta thấy một số thứ sau đây: Các biến a và c bình đẳng. Đẳng thức xảy ra khi a=c=b=1. Chú ý a+c=3−b nên VT=(3−b)(1+b), do đó ta sẽ đánh giá VP theo b.
  22. Do đó ta sử dụng 2 đánh giá 2 a c (3 b )2 ac . 2 4 22 4 2 2 (a c ) (3 b ) 2ac ( a c ) 2 4 Bài toán sẽ được chứng minh nếu (3 b )4 (3 b ) 2 (3 b )(1 b ) b b ( b2 1) hay 8(1 b ) b (3 b )3 2 b ( b 2 1)(3 b ). 8 4 Chú ý b∈(0;3), bất đẳng thức trên có thể chứng minh bằng biến đồi tương đương hoặc khảo sát hàm số. Bài 37: Cho các số 0<x , y , z , t<1 và x+y+z+t=1.Tìm giá trị nhỏ nhất của biểu thức: x y z t T 1 x 1 y 1 z 1 t Áp dụng bất đẳng thức BCS ta có: x y z t x y z t T 1 x 1 y 1 z 1 tyxtztxtxyxyz x2 y 2 z 2 t 2 xyxt()()()() yztx ztxy txyz ()x y z t 2 2(xy yz zx xt yt zt ) Ta lại có: 4(x t )( y z ) ( x t )2 ( y z ) 2 xy yz zx xt yt zt( x t )( y z ) yz xt 4 (xyzt )2 2( xtyz )( ) 3( xyzt ) 2 4 8 4 Từ đó suy ra: T 3 Cách 2 x y z t 1 1 1 1 T 4 1 1 1 1 1 x 1 y 1 z 1 t 1 x 1 y 1 z 1 t Áp dụng bất đẳng thức quen thuộc sau: 1 1 4 , a , b 0 a b a b Lúc đó 4 4 16 16 T 4 2 x y 2 z t 4 ( x y z t ) 3 16 4 Từ đó : T 4 3 3
  23. 1 Dấu đẳng thức xảy ra khi và chỉ khi : x y z t 4 Bài 38: Cho a, b , c [0;2] và a b c 3 .Tìm giá trị lớn nhất và giá trị nhỏ nhất của : 2 2 2 P a 2 b 3 c 2 a 24 c 2060 Ở bài toán này ta thấy trong biểu thức P có gì đó bất thường là vì biến b chỉ xuất hiện một lần duy nhất nên ta có thể xem P là một hàm số theo biến b và a ; c là các đại lượng cố định . Thật vậy : P f( b ) 2 b2 a 2 3 c 2 2 a 24 c 2060,  b [0 ; 2] Ta có : f ( b ) 4 b 0,  b (0 ; 2) Do đó hàm số y=f(b) đồng biến b [0 ; 2] Nên ta có : f(0) f ( b ) P f (2) Vậy ta sẽ đi xét hai bài toán nhỏ sau : Bài 1. Vì a+b+c=3→c=3−b−a Từ đó ta có : f(0) (3 c )2 3 c 2 2(3 c ) 24 c 2060 4 c 2 28 c 2063 P Tức là lúc này ta đi tìm c sao cho f( c ) 4 c2 28 c 2063 P ,  c [0 ; 2], (1) Xét hàm số f( c ) 4 c2 28 c 2063,  c [0 ; 2] Có f () c 8 c 28 0,  c [0;2] Vậy hàm số f(c) nghịch biến ∀c∈[0 ; 2] Từ (1) ta có : f() c P ,  c [0;2] min() f c f (2) P ,  c [0;2] P 2023 Vậy giá trị nhỏ nhất của P=2023 đạt được khi a=1 ;b=0 ; c=2 Bài 2. Vì a+b+c=3→c=3−b−a Từ đó ta có : f(2) (1 c )2 3 c 2 2(1 c ) 24 c 2068 2067,  c [0 1] Mà P f(2),  c [01] P 2067,  c [01] Do đó giá trị lớn nhất của P=2067 đạt được khi a=1 ; b=2 ; c=0. 1 2 3 Bài 39: Cho a,b,c 0 và 21ab 2 bc 8 ca 12 .Tìm giá trị nhỏ nhất của: P a b c
  24. Ở bài toán này ta quan sát thấy biểu thức của P chứa các phân số và điều kiện lại chứa các tích số nên ta cần phải chuẩn nguyên về một đại lượng phân số để dể nhìn.Ở đây chúng ta có a ; b; c>0 nên trong điều kiện ta chia hai vế của nó cho abc>0 ta được: 21 2 4 12 (1) c a b abc Tới đây ta chú ý rằng ta có : 12=2.1.2.3 ; 21=7.3 ; 2=2.1 ; 8=4.2 , các hệ số này có sự gắn bó mật thiết với các hệ số trong biểu thức P. Nên ta có thể đẩy điều kiện (1) về dạng: 1 2 3 1 2 3 2 4 7 2 . . a b c a b c Tới đây thì việc đổi biến sẽ làm cho bài toán có hình thức dể nhìn là điều chúng ta ưu tiên cho bài toán .Đặt : 1 2 3 x ; y ; z ( x ; y ; z 0) a b c Lúc này điều kiện bài toán trở thành : 2x 4 y 7 z 2 xyz Và biểu thức P x y z Tới đây từ điều kiện bài toán ta đi đến : 2x 4 y 2x 4 y z (2 xy 7) z 2xy 7 2xy 7 Lúc đó ta có : x y z x y (2) 2x 4 y Tới đây tham vọng của chúng ta là kéo bài toán về chỉ còn theo một biến để tiện việc đánh giá bằng những kĩ thuật cơ bản.Điều đó dẫn đến ta phải cố gắng biến đổi (2) về một biến duy nhất cũng có nghĩa rằng chúng ta cần khử được hết đại lượng 2xy−7 , mà đại lượng này ở dưới mẫu cho nên ta cần nghỉ ngay đến việc sử dụng bất đẳng thức AM−GM Ta có : 2 14 14 2x (2 xy 7) 2 x 2xy 7 2 xy 7 7 11 2 xy 7 x y x x x x x 2x 4 y 2 x 2 xy 7 2 x 2 x 2 xy 7 14 2x 11 2xy 7 11 7 x 2 .x x 2 1 2x 2 x 2 xy 7 2 x x2 11 7 Vậy ta có : P x 2 1 x x 2 2 7 7 7 7 Theo bất đẳng thức BSC ta có : 3 37. (97)1 161 2 2 x x x x
  25. Suy ra 7 7 1 7 2 1 2 4 1 2 3 x x 2 x Do đó 11 1 7 3 9 3 9 15 P x 3 x 2 x . 2x 2 x 2 x 2 x 2 Dấu đẳng thức xảy ra khi và chỉ khi : 3 7 x 7 x2 9 1 a x 3 3 14 5 4 2x y b 2xy 7 2 5 x z 2 3 2x 2 xy 7 c 2 2x 4 y z 2xy 7 Bài 40: Cho các số x,y,z thay đổi nhưng luôn thỏa mãn điều kiện: x2 y 2 z 2 1Hãy tìm GTLN và GTNN của biểu thức: A x y z xy yz zx Để ý rằng các bộ số x2 y 2 zx 2 ,, yzxy yz zx đều có mối liên hệ với nhau thông qua hằng đẳng thức (xyz )2 x 2 y 2 z 2 2( xyyzzx ) Vậy ta làm như sau: Từ giả thiết x2 y 2 z 2 1ta có 3 3(x2 y 2 z 2 ) ( x y z ) 2 do đó 3 x y z 3 Mặt khác (xyz )2 x 2 y 2 z 2 2( xyyzzx ) 1 2( xyyzzx ) t2 1 Do đó: xy yz zx với t x y z 2 t 2 1 Vậy A t với 3 t 3 2 t 2 1 Đến đây chỉ việc xét hàm số f() t t 2 x y z t 0 Bài 41: Cho 4 số x,y,z,t thay đổi thỏa mãn điều kiện: 2 2 2 2 Hãy tìm GTLN, GTNN của biểu x y z t 1 thức: A xy yz zt tx
  26. Từ biểu thức của cần tìm giá trị lớn nhất và nhỏ nhất của bài toán và điều kiện x2 y 2 z 2 t 2 1 cho phép ta nghĩ ngay đến bất đẳng thức BSC. Thật vậy ta có : |xy yz zt tx | x2 y 2 z 2 t 2 1 Từ đó :|T | 1 Vậy : T 1 1 1 Tới đây ta kết luận giá trị nhỏ nhất của A=−1 khi x z ; y t 2 2 Mặt khác ta để ý thấy rằng nếu trong biểu thức đã cho ta sử dụng nhóm hạng tử thì ta sẽ có một điều khá thuận lợi.Đó là : A=x(y+t)+z(y+t)=(x+z)(y+t) Lại từ giả thiết : x+y+z+t=0⇒x+z=−(y+t) Vậy : A ( x z )2 0 1 1 Do đó giá trị lớn nhất của A=0 đạt được khi x y ; z t 2 2 (a b c )2 1 a 3 b 3 c 3 a 2 b 2 c 2 Bài 42: Chứng minh rằng nếu a,b,c>0 thì: 2 2 2 4 a b c2 abc ab bc ca Ta sẽ chứng minh bất đẳng thức mạnh hơn là 2(a b c )2 1 a 3 b 3 c 3 a 2 b 2 c 2 2 2 2 7 a b c2 abc ab bc ca 2(a b c )2 4( ab bc ca ) Do 2 nên bất đẳng thức trên có thể viết lại thành a2 b 2 c 2 a 2 b 2 c 2 4(abbcca ) 1 abc3 3 3 a 2 b 2 c 2 2 2 2 5 a b c2 abc ab bc ca Hay 8(abbcca ) a3 b 3 c 3 a 2 b 2 c 2 10 * a2 b 2 c 2 abc abbcca Để bài toán trở nên đơn giản hơn, ta nên tìm cách giảm bớt số đại lượng khác nhau. ab bc ca a2 b 2 c 2 Cụ thể là ở đây, ta sẽ thử đánh giá thế nào đó để đưa đại lượng a2 b 2 c 2 ab bc ca
  27. Điều này gợi cho ta ý tưởng sử dụng bất đẳng thức AM-GM dạng x y 2 xy Thật vậy, dễ thấy ab bc ca a2 b 2 c 2 2 a2 b 2 c 2 ab bc ca Suy ra 8(ab bc ca ) 8( a2 b 2 c 2 ) 16 a2 b 2 c 2 ab bc ca a3 b 3 c 39( a 2 b 2 c 2 ) Sử dụng đánh giá này, ta sẽ quy về việc chứng minh 6 abc ab bc ca a3 b 3 c 3 a 2 b 2 c 2 Tương đương với 3 9 1 abc ab bc ca Hay a3 b 3 c 3 3 abc 9( a 2 b 2 c 2 abbcca ) abc ab bc ca Sử dụng hằng đẳng thức quen thuộc a3 b 3 c 33 abc ( abca )( 2 b 2 c 2 abbcca ) Ta suy ra bất đẳng thức trên có thể viết lại thành 2 2 2 a b c 9 (a b c ab bc ca ) 0 abc ab bc ca 1 Do abcabbcca2 2 2[( ab ) 2 ( bc ) 2 ( ca ) 2 ] 0 2 a b c 9 Nên chứng minh sẽ hoàn tất nếu ta chỉ ra được 0 abc ab bc ca Hay (a b cab )( bc ca ) 9 abc Điều này là hiển nhiên đúng vì theo bất đẳng thức AM-GM bộ ba số, (a b c )( ab bc ca ) 33 abc .3 a2 b 2 c 2 9 abc Bài toán được giải quyết hoàn toàn. Dấu đẳng thức xảy ra khi và chỉ khi a=b=c>0 Bài 43: Cho các số thực x , y thỏa điều kiện x2 9 y 2 1.Tìm giá trị lớn nhất và giá trị nhỏ nhất của biểu thức sau: (x 1)2 3(2 xy 1) (3 y 1) 2 T x 3 y 1 Tư tưởng của bài này chỉ là dùng phương pháp khảo sát hàm số thôi. Ta có:
  28. x2 2 x 1 6 xy 3 9 y 2 6 y 1 ( x 3 y ) 2 2( x 3 y ) 5 T x 3 y 1 x 3 y 1 (x 3 y 1)2 4 4 x 3 y 1 x 3 y 1 x 3 y 1 Từ điều kiện ta có:(x 3 y )2 6 xy 1 Suy ra (x 3 y )2 ( x 3 y ) 2 1 (x 3 y )2 6 xy ( x 3 y ) 2 2 2 (x 3 y )2 2 2 1 x 3 y 1 2 1 Đặt t x 3 y 1; 2 1 t 2 1 4 Ta sẽ khảo sát hàm số: f() t t t Bài 44: Chứng minh rằng với mọi t ∈ [−1;1], ta có: 1 t 1 t 1 1 t2 2 t 2 Bài toán này có nhiều hướng tiếp cận để chứng minh. Ở đây ban đầu nhìn nhận từ điều kiện bài toán ta thấy các biểu thức trong căn đều đã được xác định nên ta có thể nghỉ ngay đến việc chứng minh trực tiếp bài toán.Ta chia bài toán thành hai bài toán nhỏ sau : Bài 1. Chứng minh : 1 t 1 t 1 1 t2 (1) Ta có (1) luôn đúng. Thật vậy bình phương hai vế của (1) ta được : 221 t2 221 t 2 t 2 0 t 2 (lu«n ®óng) Vậy (1) được chứng minh hoàn tất. Bài 2. Chứng minh : 1 1 t2 2 t 2 (2) Từ điều kiện giả thiết : 1 t 1 Ta có 0 1 t2 1 1 t 2 1 t 2 Từ đó 1 1 t2 1 1 t 2 1 1 t 2 2 t 2 Vậy (2) được chứng minh hoàn tất. Do đó bài toán đã giải quyết hoàn toàn. 1 1 1 Bài 45: Cho ba số thực dương x,y,z thỏa mãn : x2 y 2 z 2 1 . Chứng minh rằng: (x y z ) 2 3 x y z 1 1 1 (x y z ) 4 3 x y z
  29. Trong hai bất đẳng thức trên, ta sẽ chứng minh bất đẳng thức mạnh hơn là: 1 1 1 x y z 4 3 x y z Đây là một bài tập khá cơ bản về kĩ thuật chọn điểm rơi. Thoạt nhìn, ta muốn ghép cặp các hạng tử lại để sử dụng bất đẳng thức AM-GM nhằm mục đích triệt tiêu biến số, để thu được hằng số ở VP. 1 1 1 1 Tuy nhiên, nếu đánh giá luôn x 2 thì ta sẽ suy ra x y z 6 x x y z Nhưng vì sao lại là 6 mà không phải 4 3 ? Đơn giản vì đánh giá trên đã không "bảo toàn" được dấu bằng của bài toán. Do đó, ta cần chỉnh lại hướng đi một chút để có lời giải chính xác: 1 1 1 Dễ thấy dấu đẳng thức xảy ra khi x y z Khi ấy, 3x Sử dụng AM-GM, ta có 3x 2 3 3 x x Thiết lập hai bất đẳng thức tương tự rồi cộng lại, ta thu được 1 1 1 3(x y z ) 6 3 x y z Dễ thấy chứng minh sẽ hoàn tất nếu ta chỉ ra được rằng 2(x y z ) 2 3 hay x y z 3 Điều này là hiển nhiên đúng vì theo bất đẳng thức Bunhiacopxki, x y z 3( x2 y 2 z 2 ) 3 Bài toán được giải quyết hoàn toàn Cách 2 Đặt t=(x+y+z) từ gt ta có t 3( x2 y 2 z 2 ) 3 1 1 1 9 9 9 Mặt khác Dẫn đến A t x y z x y z t t 3 6 Áp dụng bđt AM-GM ta có ngay A ( t ) 2 3 2 3 4 3 t t 1 Đẳng thức xảy ra khi x y z 3 1 1 1 Bài 46: Cho a,b,c không âm, đôi một phân biệt. Tìm min: P ( a2 b 2 c 2 )( ). ()()()a b2 b c 2 c a 2
  30. 1 21 2 2a 2 a t khi đó dễ thấy t>2. Mặt khác, a 2 t 2 vµ 2 Đặt a a( a 1) t 2 2 Do đó, bài toán quy về tìm Min của f( t ) t2 2 t 2 3 5 f t f Tính đạo hàm ra, ta nhận thấy min () 2 Bài 47: Cho a,b,c dương thỏa mãn a3 b 3 c 3 .Chứng minh rằng: a2 b 2 c 2 6 c a c b . Bài toán này tương đương với bài toán sau: " cho x,y>0 và x3 y 3 1 Chứng minh rằng: x2 y 2 1 6 1 x 1 y Áp dụng bất đẳng thức AM−GM ta có: 3 x y 8 x3 y 3 1 x y 3 4 4 5 x y 3 1 x3 y 3 1 xy 3 x y Thay vào ta được bất đẳng thức cần chứng minh là: 3 28 x y 8 8 2 x y 7 6 x y 5 x y x y 1 0 3 x y 5 Bất đẳng thức cuối luôn đúng.■. a4 b 4 c 4 3 Bài 48: Cho a,b,c>0 thỏa mãn a+b+c=3. Chứng minh . (bcbc )(2 2 )( caca )( 2 2 )( abab )( 2 2 )4 Mình tạm gọi vế trái của bất đẳng thức ban đầu là VT. Lưu ý chúng ta sẽ sử dụng giả thiết a+b+c=3 rất nhiều lần. Ta để ý rằng bất đẳng thức cần chứng minh có xuất hiện bậc 4 ở tử (tức là hai lần bình phương), như vậy ta nghĩ tới dạng sau của bất đẳng thức Cauchy Schwarz (hay còn gọi là Bunhiacopskip cho quen thuộc) m2 n 2 p 2() m n p 2 . x y z x y z (chứng minh bất đẳng thức này chỉ cần một lần dùng Cauchy Schwarz là được) Tuy nhiên, nếu ta để nguyên và áp dụng Cauchy Schwarz như sau 2 2 2 2 a2 b 2 c 2 a 2 b 2 c 2 VT . (bcbc )(22 )( caca )( 22 )( abab )( 22 )( bcbc )( 22 )( caca )( 22 )( abab )( 22 ) Như thế, ta cần chứng minh
  31. 2 2 2 2 a b c 3 . (bcbc )(2 2 )( caca )( 2 2 )( abab )( 2 2 )4 Ta thấy rằng, mẫu số của phân thức bên trái sau khi khai triển sẽ có đại lượng a3 b 3 c 3 mà theo kinh nghiệm, mẫu số mà có bậc quá cao thì đánh giá sẽ rất khó khăn (trong trường hợp này, vì khi bậc của cái nào đó quá cao thì giá trị của "chúng" thường là lớn, mà "chúng" ở dưới mẫu và đánh giá đang là ≥ bởi vậy ta cần một cái mẫu "nhỏ" hơn để nguyên phân thức của chúng ta có giá trị lớn). Bởi vì thế, ta "tạm" không đi theo hướng này. Như vậy, ta sẽ thử áp dụng tư tưởng "làm cho mẫu số có bậc nhỏ" sau khi áp dụng Cauchy Schwarz. Muốn làm điều đó, chúng ta chỉ cần để ý một điều đơn giản sau: 2 a2 a4 b2 c 2 . (b c )( b2 c 2 ) b c Tương tự hai đại lượng còn lại. Và sau khi áp dụng Cauchy Schwarz, ta sẽ thấy ngay lợi ích của phép biến đổi trên. 2 2 a2 b 2 c 2 a 2 b 2 c 2 bccaab222222 bccaab 222222 VT . b c c a a b 6 a2 b 2 c 2 3 Ta chỉ cần chứng minh rằng . b2 c 2 c 2 a 2 a 2 b 2 2 Như vậy ta đã đưa về bất đẳng thức đơn giản hơn (như ở đầu bài đã đề cập) (mặc dù có chứa căn nhưng dù gì trông nó cũng gọn hơn). Ta lại thấy tử số có xuất hiện bình phương, áp dụng Cauchy Schwarz là một điều dễ liên tưởng tới. Nhưng nếu áp dụng trực tiếp: a2 b 2 c 2( a b c ) 2 9 . bc22 ca 22 ab 22 bccaab 222222 bccaab 2222 22 Ta sẽ thấy ngay đến đây ta không thể làm gì tiếp, vì: áp dụng bất đẳng thức quen thuộc 2(x2 y 2 ) x y ta được 2 2 2 2 2 2 b c c a a b b c c a a b 3 2. 2 2 2 Nên 9 3 . b2 c 2 c 2 a 2 a 2 b 2 2 Vậy cách Cauchy Schwarz như trên là không hiệu quả.
  32. Ta áp dụng một kĩ năng khác để sử dụng bất đẳng thức Cauchy Schwarz thêm hiệu quả: "nhân tử số với chính nó rồi áp dụng Cauchy Schwarz". Cụ thể, ta có a2 b 2 c 2()()()() a 22 b 22 c 22 a 2222 b c . bc22 ca 22 ab 22 abcbcacab 222222222 abcbcacab 222222222 Vậy ta cần chứng minh (a2 b 2 c 2 ) 2 3 . a2 b 2 c 2 b 2 c 2 a 2 c 2 a 2 b 2 2 Những bước đánh giá sau nhằm để "đưa về những đại lượng quen thuộc", mong các bạn tham khảo. Đánh giá ở mẫu số, áp dụng Cauchy Schwarz một lần nữa, ta được abcbcacab222222222 aabc 2222()()() bbca 2222 ccab 2222 222222 222 222 222222222 abcabc ( ) bca ( ) cab ( ) 2 abcabbcca . Vậy ta có ()()a2 b 2 c 2 2 a 2 b 2 c 2 2 . a2 b 2 c 2 b 2 c 2 a 2 c 2 a 2 b 2 2 a2 b 2 c 2 a 2 b 2 b 2 c 2 c 2 a 2 Ta chỉ cần chỉ ra được (a2 b 2 c 2 ) 2 3 . 2 a2 b 2 c 2 a 2 b 2 b 2 c 2 c 2 a 2 2 Tương đườn với abc222 abc 222 3 abbcca 222222 . Đến đây, ta chỉ cần có đánh giá đơn giản là hai bất đẳng thức sau, ta sẽ suy ra điều phải chứng minh a b c abc222 3, abc 222 3 abbcca 222222 . 3 Tóm lại, bất đẳng thức đã được chứng minh hoàn toàn chỉ bằng mỗi bất đẳng thức Cauchy Schwarz. ■ Bài 49 Tìm x∈R để biểu thức sau đạt giá trị lớn nhất P cos x (sin x sin2 x 2). 3sin2x cos 2 x 3cos 2 x sin 2 x 2 Ta có: 3P 3sin.cos x x 3cos.sin x2 x 2 3 2 2 Vậy: P 3 Dễ thấy khi x 2 k thì P 3 6
  33. Bài 50: Cho 3 số thực dương x,y,z thoả mãn điều kiện: x + y + z = xyz. Chứng minh rằng: xy yz zx 3 x2 1 y 2 1 z 2 1 Trước hết ta sẽ thuần nhất hai vế, thật vậy bất đẳng thức đã cho tương đương với xy yz zx () x y z x y z x y z x y z 3 x2 . 1 y 2 . 1 z 2 . 1 xyz xyz xyz xyz Ta có các biến đổi sau đây xyx( y ) yzyz ( ) zxzx ( ) 3 xyz x y yz zx VT 3 xyz z x y xyyz yzzx zxxy VP z x x y y z Nên bất đẳng thức đã cho tương đương với xyyzzx xyyz yzzx zxxy z x y z x x y y z Đây là một kết quả hiển nhiên theo bất đẳng thức cơ bản a2 b 2 c 2 ab bc ca. Bài toán được chứng minh xong .□ Bài 51: Cho a,b,c là các số thực thỏa mãn: a − 2b + 3c = 16. Tìm giá trị nhỏ nhất của : P 2( a2 b 2 c 2 ) 4( a b c ) 15 Ta viết lại biểu thức phía dưới như sau: P 9 (a 1)2 ( b 1) 2 ( c 1) 2 2 2 9 P (a 1)2 ( b 1) 2 ( c 1) 2 0 2 2 P 9 Ta thấy phương trình trên có dạng phương trình mặt cầu:(x 1)2 ( y 1) 2 ( z 1) 2 (*) 2 2 và biểu thức: a−2b+3c=16 có dạng của phương trình mặt phẳng: x−2y+3z=16. P 9 Ta quy về bài toán: Cho mặt cầu có tâm I(1;1;1) và bán kính R2 và mặt phẳng(Q): x−2y+3z=16. Tìm giá trị 2 2 P nhỏ nhất để mặt cầu (∗) và mặt phẳng (Q) cắt nhau. Ta có: d( I ,( Q )) 14 P 9 Do đó RRP 14 2 14 14 37 2 2 Đẳng thức xảy ra khi và chỉ khi (a,b,c)=(2;−1;4) Vậy minP=37
  34. a3 b 3 c 3 abc 8 Bài 52: Cho các số dương a,b,c. Tìm giá trị nhỏ nhất của biểu thức P . a b c a3 b 3 ab( a b ) 8 6( a b ) c3 b 3 cb( c b ) 8 6( c b ) a3 c 3 ac( a c ) 8 6( a c ) Chứng minh BDT trên như sau: (abab )(( )3)2 ababab ( )86( ab ) (a b )3 2 ab ( a b ) 8 6( a b ) Thế (a b )2 4 ab vào thì cần CM ()()a b3 a b 3 (a b )3 8 8 6( a b ) (đúng) 2 2 Vì đây chính là cosi 3 số Thực hiện lần lượt với 2 cơ số còn lại rồi cộng 3 BDT trên ta thu được 222a3 b 3 cababbcbccaca 3 ( )( )( )2412( abc ) 222a3 b 3 c 3 abab ()()()24 bcbc caca 12 a b c Kết hợp với BDT mà bạn trên đã nêu ra a3 b 3 c 3 3 abc aba ( b ) bcbc ( ) cac ( a ) Ta sẽ có được min của P=4 khi a=b=c=1 cách này gần như là cách của bạn trên nhưng mà làm với 2 cơ số rồi cộng lại. Cách 2 Hình thức biểu thức P gợi ý rõ ràng cho ta rằng min của nó tìm được bằng AM_GM. Thật vậy, áp dụng BDT AM- GM, và Schur bậc 3 a3 b 3 c 3 3 abc aba ( b ) bcbc ( ) cac ( a ) Ta có 27 6.75(a3 b 3 c 3 ) 6.75 abc 54 a 3 b 3 c 3 6 abc 3[ ab ( a b ) bc ( b c ) ca ( c a )] 54 P 4 a b c a b c (a b c )3 27 27 AM GM 27. a b c Bất đăng thức Schur bậc ba ở trên dễ dàng được viết lại là c(a+b−c)2+c(c−a)(c−b)≥0. Nó đúng vì chỉ cần giã sử c là số nhỏ nhất trong ba số a,b,c mà thôi. Vậy giá trị nhỏ nhất của P là 4 khi a=b=c=1.
  35. Bài 53: Cho hai số không âm a và b thỏa mãn: a b 2 10 . Chứng minh rằng : (1 a4 )(1 b 4 ) 101 Giải Bài 54: Cho a,b,c không âm thỏa mãn a b c 3 Tìm giá trị lớn nhất của P a2 abb 2 b 2 bcc 2 c 2 caa 2 Giải Không mất tính tổng quát của bài toán, ta có thể giả sử c=min{a,b,c}, khi đó ta có b2 bc c 2 c() c b b 2 b 2 c2 ca a 2 c() c a a 2 a 2 Từ các đánh giá này ta thu được P a2 b 2() a 2 ab b 2 Mặt khác theo bất đẳng thức AM-GM thì 4 3ab 3 ab a2 b 2()() a 2 ab b 2 a 2 ab b 2 9 2 2 3 3ab 3 ab a2 ab b 2 4 2 2 9 3 4 4 (a b )6 ( a b c ) 6 12 35 3 5 Đẳng thức xảy ra khi a=2,b=1,c=0. Vậy GTLN của biểu thức cần tìm là 12.□ 4 Bài 55: Cho 3 số thực dương x,y,z thỏa x + y + z = 1. Chứng minh rằng: x2 y y 2 z z 2 x 27 Giải Giả sử x=max{x,y,z}. Ta có: z2 x z 2 x z 2 x zx 2 Axyyzzxxyyz 2 2 2 2 2 xyxyz 2 2 2 2 2 zx xy()() x z z x 2 z x z x z x( y )( z x ) 4 ( y )( ) 2 2 2 2 2 3 x z x z y 2 2 2 2 4 4 . 27 27 2 1 2 1 1 2  Dấu đẳng thức xảy ra khi và chỉ khi (,,)x y z ,,0;0,, ; ,0,,  . 3 3 3 3 3 3 
  36. 1 1 1 3 Bài 56: Cho x,y,z≥0 thỏa mãn: xyz=1 . Chứng minh rằng: (1 x )2 (1 y ) 2 (1 z ) 2 4 Giải Bài 57: Cho a,b,c là các số thực dương thỏa mãn a + b + c ≤ 1. Tìm giá trị nhỏ nhất của biểu thức: 1 1 1 P 3 a3 3 b 3 3 c 3 . a3 b 3 c 3 Giải a sử dụng đánh giá quen thuộc sau đây (có thể chứng minh bằng phép biến đổi tương đương): Với x là số thực dương thì 3 3 1 1 1 x 3 x . x4 x Sử dụng đánh giá này ta có 1 1 1 1 P a b c . 3 4 a b c Mặt khác ta lại có 1 1 1 9 a b c a b c a b c a b c Và 9 1 8 a b c a b c 2 8 10. a b c a b c a b c 10 1 Từ đó suy ra P với đẳng thức xảy ra khi a b c min 3 4 3 Bài 59: Cho hai số dương x,y thỏa mãn điều kiện: x 1; y 1 và 3 x y 4xy . Tìm GTLN,GTNN của biểu 1 1 thức: P x3 y 3 3( ) x2 y 2 Giải Từ điều kiện bài toán nếu ta đặt : S x y P xy Ta có ngay được : 3S P 4
  37. Mặt khác theo cách đặt này ta có S và P ràng buộc nhau bởi điều kiện : SPSSSS2 4 0 2 3 0 0  3 Mà từ điều kiện x≥1 ; y≥1 ta suy ra S=x+y≥2 Vậy ta có : S≥3 Lại có theo điều kiện x≥1 ; y≥1 thì (x−1)(y−1)≥0⇔xy−(x+y)+1≥0⇔S−3S4+1≥0⇔S≤4 Do đó tổng kết lại ta có : 3≤S≤4 Bây giờ ta để ý từ điều kiện bài toán : 1 1 4 3(x y ) 4 xy x y 3 Tiếp tục áp dụng hằng đẳng thức quen thuộc ta có : 2 3 3 3 1 1 1 1 2 x y ( x y ) 3 xy ( x y ) ; x y x y xy 9 8 16 Từ đó ta có PSS 3 4S 3 9 8 16 f( S ) S3 S , 3 S 4 4S 3 9 8 3 8 2 f( S ) 3 S S 2 3 S S 2 0 2SS 2 Vậy hàm số y=f(S) đồng biến ∀S∈[3 ; 4 f(3) f ( S ) f 4 Vậy 113 Giá trị nhỏ nhất của biểu thức P bằng f (3) đạt được khi và chỉ khi : 12 S x y 3 9 3 P xy x y 4 2 x 1; y 1 94 Giá trị lớn nhất của biểu thức P bằng f (4) đạt được khi và chỉ khi : 3
  38. x 1 S x y 4 y 3 P xy 3 x 1; y 1 x 3 y 1 2abc 2 bc 2 ca Bài 60: Cho a,b,c>0 thỏa mãn a + b + c = 1 . Tìm giá trị lớn nhất của biểu thức P a2 b 2. a b b c c a Giải Bài 61:Cho a,b,c là các số thực dương thỏa mãn:a+b+c=2. Tìm GTLN của: ab bc ca M 2c ab 2 a bc 2 b ca Giải Sử dụng bất đẳng thức AM-GM, ta có ab bc ca ab bc ca M 2cab 2 abc 2 bca ()()()()()() cacb abac abbc 1 ab ab bc bc ca ac ( ) 1 2 cacbabcaabbc 2 Vậy GTLN của biểu thức là 1 khi a b c 3 a2 b 2 c 24( a b ) 2 Bài 62: Cho các số thực dương a,b,c. Chứng minh rằng: a b c b c a a b c Bất đẳng thức trên tương đương với a2 b 2 c 24( a b ) 2 a b c b c a a b c 2 a2 a b Xét biểu thức vế trái . Để ý rằng, theo bất đẳng thức AM-GM thì b 2 a tương đương với 0 b b Từ đó ta suy ra phân tích: abc2 2 2 a 2 b 2 c 2 ()()() abbcca 2 2 2 a b c b2 a c 2 b a 2 c b c a b c a b c a Vậy bất đẳng thức cần chứng minh có thể viết lại thành
  39. (a b )2 ( b c ) 2 ( c a ) 2 4( a b ) 2 b c a a b c Một cách tự nhiên, ta liên tưởng ngay đến bất đẳng thức Cauchy Schwarz dạng phân thức. Thật vậy, sử dụng Cauchy Schwarz, ta có ()()()()b c2 c a 2 b c c a 2 a b 2 c a c a c a Dễ thấy chứng minh sẽ hoàn tất nếu ta chỉ ra được 2 1 1 4 (a b ) 0 b c a a b c Điều này là hiển nhiên đúng vì một lần nữa, sử dụng bất đẳng thức Cauchy Schwarz, suy ra 1 1 4 Bài toán được giải quyết hoàn toàn b c a a b c 9 Bài 63: Cho a,b,c là ba số thực không âm có tổng bằng 3. Chứng minh: a ab 2 abc 2 Ý tưởng. Ta muốn quy bất đẳng thức đã cho về dạng một biến đơn giản hơn. Vấn đề biến đó là biến nào ? Sử dụng một chút cảm giác, tôi tin ai cũng sẽ nghĩ tới biến a. Vậy công việc của ta là làm thế nào đó để đưa cái thằng ab+2abc về biểu thức chỉ chứa biến a. Và cuối cùng, ta đi đến lời giải. Lời giải. x y 2 Sử dụng bất đẳng thức AM-GM dạng xy ta có 4 2 2 1 1 b c 3 a 2 1 2 2 a (7 2 a ) b 2 abc 2 a . b c 2 a . 2 a . 2 4 4 8 Do đó, chứng minh sẽ hoàn tất nếu ta chỉ ra được a(7 2 a )2 9 a 8 2 Thật vậy, khai triển và rút gọn, bất đẳng thức trên trở thành (4 a )(2 a 3)2 0 3 1 Luôn đúng với 0≤a≤3. Bài toán được giải quyết hoàn toàn. Dấu đẳng thức xảy ra khi và chỉ khi(a , b , c ) ,1, 2 2
  40. Bài 65: Cho a,b,c là các số thực không âm có tổng bằng 1. Chứng minh rằng: 4(a3 b 3 c 3 ) 15 abc 1 Bất đẳng thức đã cho tương đương với: [(a b )3(3 ab a b ) c 3 ]15 abc 1 4[(1 c )3(1 3 ab c ) c 3 ]15 abc 10 3(5abc 44)312 c cc 122 0 3(9 cab 4) 312 cc 12 2 0 (a b )2 (1 c ) 2 Ta đặt: x ab(0 x ) 4 4 1 c 2 Rõ ràng hàm số: f x 3 9 c 4 x 3 12 c 12 c2 là một hàm bậc nhất với biến x 0; 4 Ta có: f(0) 212 c 12 c2 2(12) x 2 0 (1 c )2 3(9 c 4)(1 c ) 2 f( ) 31212 c c2 3(31)0 c c 2 4 4 1 c 2 Suy ra: f(x)≥0 với mọi x 0; Ta có điều phải chứng minh. 4 1 1 Dấu ′′=′′ xảy ra khi: a b c hoặc: a b , c 0 và các hoán vị. 3 2 cách 2 1 2 Giả sử: c=min{a;b;c} 0 c a b 1 3 3 Đặt: f( a ; b ; c ) 4( a3 b 3 c 3 ) 15abc 1 Ta có: a b a b15 c ( a b )2 f( ; ; c ) ( a b )3 4 c 3 1 2 2 4 Ta có: a b a b15 c ( a b )2 f( a ; b ; c ) f ( ; ; c ) 4( a3 b 3 ) ( a b ) 3 15 abc 2 2 4 15c ( a b )2 15 c 3(a b )( a b )2 ( a b ) 2 [3( a b ) ] 0 4 4 15c 2 15 3 Đúng vì: 3(a b ) 3. 0 4 3 3.4 4 Suy ra ta đưa được về bài toán 2 biến là: Chứng minh rằng: f(;;)4(2 t t c t3 c 3 )15 t 2 c 10
  41. a b Với: 2t c 3; t Bạn rút: c=3−2t, rồi thế lên trên ta thu được hàm số với biến t, từ đó khảo sát hàm số nữa 2 là xong! Bài 66: Chứng minh rằng với mọi số thực dương a,b,c, ta luôn có a2 b 2 c 2 ab bc ca ab bc ca a2 bc ca b 2 ca ab c 2 ab bc Lời giải Sử dụng bất đẳng thức Cauchy−Schwarz ta có: 2 2 ab ab b bc ca ab b bc ca a2 bc ca a2 bc ca b 2 bc ca ab bc ca 2 Tương tự, ta có: 2 bc bc b ca ab b2 ca ab ab bc ca 2 2 ca ca c ab bc c2 ab bc ab bc ca 2 Cộng vế theo vế 3 bất đẳng thức trên, ta được: 2 2 2 ab bc ca ab b bc ca bc b ca ab ca c ab bc a2 bc ca b 2 ca ab c 2 ab bc ab bc ca 2 Do đó ta chỉ cần chứng minh 2 2 2 ab b bc ca bc b ca ab ca c ab bc a2 b 2 c 2 ab bc ca 2 ab bc ca Hay tương đương ab bc ca a2 b 2 c 2 ab b 2 bc ca bc b 2 ca ab ca c 2 ab bc ab3 bc 3 ca 3 abca b c a2 b 2 c 2 a b c c a b Bất đẳng thức cuối đúng theo Cauchy−Schwarz. Bài toán được chứng minh xong. Đẳng thức xảy ra khi và chỉ khi a=b=c.
  42. 1 x2 1 y 2 1 z 2 Bài 67: Cho x,y,z là các số thực lớn hơn −1. Chứng minh rằng: 2 1 y z2 1 z x 2 1 x y 2 ()a b c2 a b b c c a Bài 68: Chứng minh rằng nếu a,b,c>0 thì: abbcca ac ba cb a ab 3 abc a b a b c Bài 69: Chứng minh rằng với mọi a,b,c>0, ta có: 3 a 3 2 3 a3 b 3 c 3 Bài 70: Cho a,b,c là các số thực dương thỏa mãn a+b+c=3. Chứng minh rằng 3 83 abc 9, 3 Ta có: abc3 3 3( abc ) 3 3( abbcca )( )( ) 27 24 abc a3 b 3 c 3 27 24 abc Suy ra ta có: 3 83abc 3 8 3 abc 3 9 8 abc 8 3 abc 3 3 Tới đây chỉ còn mỗi một biến:abc nên ta nghĩ tới phương pháp hàm số để khảo sát hàm một biến. Đặt: t=abc. Khi đó ta có: f( t ) 3 9 8 t 8 3 t a b c Ta có:t abc ( )3 1 0 t 1 3 8 8 8 1 1 Ta lại có: f ()[] t 33 (9 8t )2 33 t 23 3 t 2 3 (9 8 t ) 2 Ta có:(9 8t )2 t 2 (9 9 t )(9 7 t ) 9(1 t )(9 7 t ) 0 f ( t ) 0 Suy ra: f( t ) f (1) 9 Suy ra ta có điều phải chứng minh. Dấu ′′=′′ xảy ra khi:a=b=c=1 Bài 71: Cho x,y,z là các số thực dương thỏa mãn xy+yz+zx=1.Chứng minh rằng: x y z 9 y(1 x2 ) z (1 y 2 ) x (1 z 2 ) 4 Để ý rằng, x2 1 x 2 xyyzzx ( xyxz )( ) nên bất đẳng thức cần chứng minh tương đương với x y z 9 yxyxz( )( )( zyzyx )( )( xzxzy )( )4 Đồng bậc hóa, ta thu được x y z 9 yxyxz( )( )( zyzyx )( )( xzxzy )( )4( xyyzzx ) Nhân cả hai vế của bất dẳng thức trên với (x+y)(y+z)(z+x), suy ra
  43. xyz( )( yzx )( zxy )9( xyyzzx )( )( ) y z x4( xy yz zx ) Tương đương với xy2 2 yz 2 2 zx 2 2 xyzxyz( ) 9 ( xyyzzx )( )( ) . xyz4 xy yz zx hay 9 (zx zy )( yx yz )( xy xz ) ()()()().*xy2 yz 2 zx 2 xyzx yz 4 xy yz zx Dễ thấy nếu đặt a=yz,b=zx,c=xy thì trở thành 9 abcabbcca2 2 2 .( abbcca )( )( ) 4(a b c ) ()x y z 3 Sử dụng bất đẳng thức AM-GM bộ ba số dạng xyz 27 Từ đánh giá trên, dễ thấy chứng minh sẽ hoàn tất nếu ta chỉ ra được rằng 2 a2 b 2 c 2 abbcca () abc 2 3 Hay 1 abc2 2 2 abbcca[( ab ) 2 ( bc ) 2 ( ca ) 2 ] 0 2 Hiển nhiên đúng. Dấu bằng xảy ra khi và chỉ khi x=y=z. 1 1 1 Bài 72: Cho a,b,c là các số dương thay đổi sao cho 1. Tìm giá trị nhỏ nhất của biểu thức a 1 b 1 c 1 a b c P . 1 1 1 a b c 1 1 1 Ta vẫn dự đoán GTNN đạt được khi: a=b=c=2.Từ giả thiết:1a+1 1 ta suy ra: a 1 b 1 c 1 1x 1 y 1 z Tồn tại các số thực dương x,y,z sao cho: ,, a 1 xyzb 1 xyzc 1 xyz y z z x x y 1 1 1 Từ đây ta có: a ,, b c .Ta cần chứng minh: a b c 4 hay là: x y z a b c y z z x x y4 x 4 y 4 z x y z y z z x x y Nhưng ta thấy bất đẳng thức này hiển nhiên đúng vì nó tương đương với:
  44. x()()() y z2 y z x 2 z x y 2 0 yzy()()() z xzx z xyx y Như thế bài toán giải quyết xong! GTNN của biểu thức đã cho là 4, đạt được khi a=b=c=2. Bài 73: Cho x,y,z là các số thực không âm thỏa mãn x3 y 3 z 3 3. Chứng minh rằng xy yz zx xyz 2 Lời giải: Không mất tính tổng quát, ta có thể giả sử z là số bé nhất trong ba số x,y,z.Từ x3 y 3 z 3 3 ta suy ra z≤1. Đặt P=xy+yz+zx−xyz ta viết P dưới dạng: P=xy(1−z)+z(x+y) Bây giờ ta tìm cách đưa biểu thức P về chỉ chứa một biến z. Sử dụng AM-GM ta có: x3 1 1 3 x , y 3 1 1 3 y , x3 y 3 4 7 z 3 Cộng hai bất đẳng thức này lại ta có: x3 y 3 4 3 x 3 y x y 3 3 4 x3 Tiếp theo ta có: x3 y 3 1 3 xy xy . 3 (4 z3 )(1 z ) z (7 z 3 ) Từ hai bất đẳng thức trên với chú ý z≤1 ta có: P 3 3 (4 z3 )(1 z ) z (7 z 3 ) Như vậy ta chỉ cần chứng minh : 2 nữa là xong, tuy nhiên ta thấy bất đẳng thức này hiển 3 3 nhiên đúng vì nó tương đương với: (z 1)2 ( z 2) 0 Vậy bài toán được chứng minh xong! Đẳng thức xảy ra khi và chỉ khi a=b=c=1. ■ 2 1 1 9 Bài 74: Cho các số thực dương x,y,z thỏa mãn x+y+z=xyz. Chứng minh rằng . 1 x2 1 y 2 1 z 2 4 xyz xxyxz( )( ) ( xyxz )( ) Để ý rằng1 x2 x 2 . Do đó, bất đẳng thức cần chứng minh tương x y z x y z yz yz zx xy 1 đương với 2 . (xyxz )( ) ( yzyx )( ) ( zxzy )( )4 Bây giờ, sử dụng bất đẳng thức AM-GM, ta có yz yz zx xz xy xy 2 , , . Cộng ba ()()xyxz xyxz ()() yzyx xy 4()()() yz zxzyzx 4() yz bất đẳng thức trên lại, ta thu được ngay yz zx xy xyxzyz 9 2 . ()()()()()()xyxz yzyx zxzy xyxz 4()4 yz
  45. Bài toán được chứng minh xong. ■ Cách 2 Ta có một cách khác nhẹ nhàng hơn như sau. Nhìn vào bài toán gợi ý cho ta sử dụng lượng giác để giải. Ta sẽ đặt: x=tanA;y=tanB;x=tanC. Với mọi tam giác ta luôn có: tanA+tanB+tanC=tanA.tanB.tanC, hay A,B,C là 3 góc của một tam giác. 2 2 Ta có: 2cos A 1 x2 1 tan 2 A Thiết lập các biểu thức tương tự, ta cần phải chứng minh: 9 2cosABC cos cos 4 Đây là một bài toán khá cơ bản, ta sẽ làm như sau: ABCBCAAA 1 9 9 2cosABC cos cos 2(1 2sin2 ) 2cos( ).cos( ) 4sin 2 2sin 2 (2sin ) 2 2 2 2 2 2 2 2 4 4 Điều phải chứng minh. x2 y 2 xy Bài 75: Cho x,y>0 thỏa mãn xy + x + y = 3. Tìm min của biểu thức: P y 1 x 1 x y Ta có: xy=3−(x+y) x2 y 2 xy( x y ) 2 3 ( x y ) Áp dụng bất đẳng thức Cauchy-Schawrz ta có: P y 1 x 1 x y x y 2 x y t2 3 Đặt t=x+y, ta có: P 1 f ( t ) t 2 t Ta sẽ tìm điều kiện t và khảo sát hàm f(t) là được. ()x y 2 Ta có:3 x y xy x y t2 4 t 12 0 t 2 4 t2 4 t 3 4 3 4 3 3 Ta có: f ( t ) 1 1 0 Suy ra: f( t ) f (2) Dấu ′′=′′ xảy ra khi: x=y=1 (t 2)2 t 2 ( t 2) 2 t 2 16 4 2 1 1 1 1 Bài 76: Cho a,b,c≥0: ab + bc + ca = 1 . Tìm Min : P a b b c c a a b c Thông thường Min của một biểu thức đối xứng ba biến đạt khi ba số bằng nhau hoặc 1 số bằng 0, hai số bằng nhau. 1 1 1 Mặt khác, coi P=f(a,b,c) thì so sánh f ,, và f(1,1,0), ta sẽ nhận thấy f(1,1,0) nhỏ hơn. 3 3 3 Do đó, Gia Cát Dự MinP=2 khi a=b=1,c=0, tức
  46. 1 1 1 1 2 a b b c c a a b c Thao tác tiếp theo, quan trọng không kém là quy bài toán về việc chứng minh bất đẳng thức đồng bậc. Sau khi đồng bậc hóa, dễ thấy ta cần chứng minh 1 1 1 1 2 a b b c c a a b c ab bc ca Nhân cả hai vế với a+b+c, ta thu được a b c2( a b c ) 2 b c c a a b ab bc ca Ta muốn đưa cái cậu Vế Trái về một cậu nào đó có họ hàng với cậu Vế Phải. Một cách tự nhiên, sử dụng bất đẳng thức Cauchy Schwarz, dễ thấy a b c() a b c 2 b c c a a b2( ab bc ca ) (a b c )2 2( a b c ) Vậy chứng minh sẽ hoàn tất nếu ta chỉ ra được 2 2(ab bc ca ) ab bc ca a b c Tới đây thấy hai vế có liên quan rồi thì tội gì không đặt x Khi đó bất đẳng thức trên trở thành ab bc ca t2 2 2t Hiển nhiên đúng theo AM-GM. Bài toán được giải quyết hoàn toàn 2 2 2 2 2 2a 2 b 2 c 9 a b c Bài 77: Cho a,b,c là các số thực dương. Chứng minh rằng 1 1 1 b c a ab bc ca Áp dụng bất đẳng thức Cauchy-Schwarz ta có: 2 2 2 2 2 2a 2 b 2 c 1 a b c 1 ( a b c )2 1 1 1 3 2( ) 3 2. b c a 3 bca 3 abbcca ()a b c 2 Đặt t ( t 3) Ta cần chứng minh: ab bc ca 1 (32) t2 9 t (32) t 2 27 t 4 t 2 15 t 90 ( t 3)(4 t 3)0 Hiển nhiên đúng, điều phải chứng minh. 3 1 1 1 Bài 78: Giả sử a,b,c là các số thực dương thỏa mãn: abc=1. Chứng minh rằng: 1 a b c 2 1 a b c Do abc=1 nên bất đẳng thức trên có thể viết lại thành1 a b c 2 1 ab bc ca Sử dụng bất đẳng thức AM-GM dạng x y 2 xy ta có:1 a b c 2 (1 a )( b c )
  47. Vậy chứng minh sẽ hoàn tất nếu ta chỉ ra được (1)()1 abc abbcca bc 1 bc (1)(1)0 b c Mặt khác, do abc=1 nên trong ba số a,b,c, luôn tồn tại ít nhất một số lớn hơn 1 và một số nhỏ hơn 1. Không mất tính tổng quát, ta có thể giả sử hai số đó là b,c để từ đó suy ra (b 1)( c 1) 0 Bài toán được giải quyết xong. Dấu đẳng thức xảy ra khi và chỉ khi a=b=c=1. Bài 79: Tìm GTNN, GTLN của hàm số: y x 4 4 x 16 x2 Bài toán này khá cơ bản vì thực ra hình thức của bài toán đã lộ rõ ý tưởng để giải quyết nó chính là đặt ẩn phụ rồi khảo sát hàm một biến ẩn phụ đó hoặc sử dụng cách tìm lớn nhất nhỏ nhất trên đoạn cho trước. Thật vậy: Điều kiện bài toán : 4 x 4 Đặt t 4 x 4 x th× 2 2 t 4 t2 8 Khi đó ta có :t2 8 2 16 x 2 16 x 2 2 Vậy bài toán ban đầu đã được đưa về bài toán tìm giá trị lớn nhất và giá trị nhỏ nhất của hàm số t2 8 f( t ) t ,  t 2 2 ; 4 2 Bài 80: Cho 2 số thực x,y thỏa: x2 y 2 4 .Tìm giá trị nhỏ nhất của biểu thức: P 5 2 x 54 2 x 14 y Lời giải: Bài này nếu mới nhìn vào ta sẽ thấy bối rối không biết bắt đầu từ đâu, tuy nhiên nếu bình tâm lại thì ta thấy rằng: P là biểu thức chứa hai dấu căn, bởi vậy "như một thói quen" ta cố đưa P vì dạng: ()()()()x a2 y b 2 x c 2 y d 2 Thật may với giả thiết x2 y 2 4 ta hoàn toàn có thể đưa về dạng trên.Thật vậy: Pxy 2 212 xxyxy 2 2 21450 (1) x 2 yx 2 (1)(7) 2 y 2 Tới đây chắc hẳn là đơn giản rồi, các bạn có thể dùng vector để đưa ngay ra đáp số, tuy nhiên ta cũng có thể tìm giá trị nhỏ nhất của P như sau: Ta thấy ngay: P y y 7 y 7 y 7 2 (x 1) 0 2 2 Đẳng thức xảy ra thì ta phải có: Chú ý với x y 4 nữa ta tìm được x=1,y= 3 .Như thế ta đi đến y(7 y ) 0 kết luận. Giá trị nhỏ nhất của P là 7, đạt được khi x=1,y= 3 . 1 1 1 3 Bài 81: Cho a,b,c là các số thực dương thỏa mãn: ab + bc + ca = 3. Chứng minh rằng: a2 1 b 2 1 c 2 1 2 Mình xin được gợi ý một phương pháp để "xơi" bài toán này đó là sử dụng bất đẳng thức quen thuộc sau 1 1 2 với ab≥1. a2 1 b 2 1 ab 1
  48. Mình xin được đóng góp lời giải mà không dùng bổ đề như anh Huyện đã nêu! Nếu ta cứ để yên biểu thức như trên thì đánh giá quả là khó khăn, như vậy ta nghĩ ngay đến việc đổi biến.Thật vậy: Đặt: x=bc,y=ca,z=ab như thế ta có: x+y+z=3.Bây giờ ta biểu diễn a,b,c theo x,y,z. yz xz xy x y z 3 Dễ thấy rằng: a2 ,, b 2 c 2 Như thế ta viết bất đẳng thức trên thành: Tới x y z x yz y zx z xy 2 đây nhiều bạn sẽ áp dụng ngay bất đẳng thức Cauchy-Schwarz như sau: x y z() x y z 2 Tuy nhiên, lời giải sẽ rơi vào bế tắc bởi ta không thể chỉ ra rằng: xyz yzx zxy x2 y 2 z 2 3 xyz (x y z )2 3 x2 y 2 z 2 3 xyz 2 Đến đây nhiều bạn sẽ nghĩ không dùng Cauchy-Schwarz được nhưng không hẳn vậy, một kinh nghiệm cho thấy với những bất đẳng thức kiểu này khi ta đẩy được về một biến thì bài toán sẽ gần như là được giải quyết, với kinh nghiệm đó ta dùng Cauchy-Schwarz cho bài này như sau: x y z() x y2 z x yz y zx z xy x2 y 2 2 xyz z xy Ta thấy: x2 y 2 2 xyz ( x y ) 2 2 xy ( z 1) kết hợp với x+y+z=3 thì ta có thể viết bất đẳng thức thành: x y z(3 z )2 z x yz y zx z xy(3 z )2 2 xy ( z 1) z xy (x y )2 (3 z ) 2 Lại để ý xy Như vậy chỉ cần z≥1 nữa thôi là có thể "đẩy" hết về biến z.Ta có được điều này khi 4 4 z=max{x,y,z}. Với những phân tích trên ta làm như sau: (x y )2 (3 z ) 2 Ta giả sử z=max{x,y,z} như thế thì z≥1.Áp dụng bất đẳng thức xy ta được: 4 4 x y z(3 z )2 z (3 z ) 2 z xyzyzxzxy (3 z )2 2 xyz ( 1) zxy (3 z )2 ( z 1) (3 z ) 2 (3 z )2 z 2 4 Với một vài biến đổi thì để chứng minh: x y z 3 2 4z 3 Ta chỉ cần chỉ ra rằng: Tuy nhiên bất đẳng thức hiển nhiên đúng x yz y zx z xy 2 z 1 z2 2 z 9 2 bởi nó tương đương với: (z 1)2 ( z 3) 0 Như thế bài toán được chứng minh! Bài 82: Xét các số thực a,b,c,d thoả mãn điều kiện a2 b 2 1, c d 3. Tìm giá trị lớn nhất của M ac bd cd Theo bất đẳng thức Bunhiacopxki, ta có:(a2 b 2 )( c 2 d 2 ) ( ac bd ) 2 Hay()()c2 d 2 ac bd 2 Từ đó suy ra, M ()| ac bd cd ac bd | cd c2 d 2 cd (3) d 2 d 2 (3)2693 d d d 2 d d 2 d
  49. (2d 3) Xét hàm f( d ) 2 d2 6 d 9 d 2 3 d trên R, ta có: f () d Lập bảng biến thiên, ta tìm 2d2 6 d 9 (2 d 3) 9 3 được maxf ( d ) 4 2 Cách 2 Áp dụng bất đẳng thức Cauchy Schwarz ta có: M ac bd cd ( a2 b 2 )( c 2 d 2 ) cd c 2 d 2 cd Thay:c=d+3 vào ta được: M 2 d2 6 d 9 ( d 2 3 d ) Tới đây ta sẽ xét hàm số: f( d ) 2 d2 6 d 9 ( d 2 3 d ) 2d 3 2(2 d 3)( d2 3 d 4) 3 Ta có: f ( d ) (2 d 3) Suy ra: f ( d ) 0 d Từ đó ta vẽ 2d2 6 d 9 2d2 6 d 9 1 2 d 2 6 d 9 2 9 6 2 3 bảng biến thiên ta dễ dàng có: f() d tại x 4 2 Bài 83: Cho các số thực dương a, b, c. Tìm giá trị nhỏ nhất của biểu thức: 2a2 2 b 2 3 c 2 4 a 2 b 2 T ()()()()()a b2 b c 2 c a 2 a b 2 b c 2 a b c Bài 84: Cho a,b,c>0 và abc=1. Chứng minh rằng: a2 b b 2 c c 2 a 2012 a b c ab bc ca Bài 85: Cho các số thực x,y thỏa mãn x2 y 2 xy 6( x y ) 11 0 . Tìm giá trị nhỏ nhất và giá trị lớn nhất của biểu thức P 2x y. Tư tưởng chính của chúng ta ở dạng này là đưa về phương trình bậc 2 và tìm điều kiện có nghiệm là ổn. Ta có: y=P−2x. Thay vào biểu thức phía trên ta được: 3x2 (6 P ) x P 2 6 P 11 0 Lấy điều kiện Δ nữa là ổn. Bài 86 :Cho a,b,c là các số thực dương thoả mãn a3 b 3 c 3 3 . Chứng minh rằng a2 b 2 c 2 1 3 3 3 b 8 c 8 a 8 Ta quan sát thấy vế trái của bất đẳng thức thì chứa căn, trong khi dấu của bất đẳng thức lại là "≤" như thế ta nghĩ đến ngay việc dùng bất đẳng thức Cauchy-Schwarz kiểu (axbycz )2 ( a 2 b 2 cx 2 )( 2 y 2 z 2 ) 2 2 2 2 a b c 3 3 3 a b c Thật vậy, áp dụng Cauchy-Schwarz ta có: ()a b c 3 3 3 b3 8 c 3 8 a 3 8 b 8 c 8 a 8 a b c 1 Kết hợp với a3 b 3 c 3 3 thì để bài toán được chứng minh ta cần chỉ ra rằng: Ta thấy b3 8 c 3 8 a 3 8 3 số hạng dưới mẫu có bậc khá lớn so với tử, như thế ta cần đánh giá để giảm bậc của nó đi, điều này làm ta nhớ đến bất đẳng thức AM-GM, dự đoán điểm rơi xảy ra khi a=b=c=1 ta sẽ áp dụng AM-GM như sau: a3 8 a 3 1 1 6 3 a 6 3( a 2) Tương tự như thế ta có: b3 8 3( b 2), c 3 8 3( c 2) Bởi vậy để chứng minh bất đẳng thức trên ta đi chứng a b c minh: 1 b 2 c 2 a 2 Đến đây ta không thấy một hướng đánh giá nào khả quan cả, như thế ta sẽ biến đổi trực tiếp.Quy đồng mẫu số, khai triển rút gọn ta thu được bất đẳng thức tương đương: ab2 bc 2 ca 2 2( a 2 b 2 c 2 ) abc 8 Đến đây ta chia ra chứng minh hai bất đẳng thức sau: a2 b 2 c 2 3 ab2 bc 2 ca 2 abc 2 Việc chứng minh hai bất đẳng thức này là khá đơn giản, mình xin dành cho bạn đọc tự hoàn thiện!
  50. a3 b 3 c 3 Bài 87: Cho a,b,c≥0 thỏa mãn a2 b 2 c 2 3.Tìm GTNN của biểu thức: P b2 1 c 2 1 a 2 1 Lời giải: a3 b 3 c 3() a 2 b 2 c 2 2 Áp dụng trực tiếp Cauchy-Schwarz ta có: P b2 1 c 2 1 a 2 1 a b 2 1 b c 2 1 c a 2 1 Bây giờ áp dụng Cauchy-Schwarz cho biểu thức dưới mẫu ta có: (ab2 1 bc 2 1 ca 2 1)( 2 abcb 2 2 2 )(11 2 c 2 a 2 1) ()a2 b 2 c 2 3 3 2 Như thế ta có: P Thay a2 b 2 c 2 3 vào ta thu được min P Đẳng thức xảy ra khi và a2 b 2 c 2 3 2 chỉ khi a=b=c=1. a3 a a 2 1 Cách 3: Áp dụng bất đẳng thức AM−GM , ta có: 2a2 a2 1 2 Thiết lập 2 bất đẳng thức tương tự rồi cộng vế theo vế, ta được: a3 b 3 c 3 a a 2 1 b b 2 1 c c 2 1 2(a2 b 2 c 2 ) 3 2 b2 1 c 2 1 a 2 1 2 2 2 aa2 1 bb 2 1 cc 2 12 aa 22 12 bb 22 12 cc 22 13( abc 222 )33 Mặt khác: 2 2 2 4 2 4 2 4 2 4 2 2 3 2 ta thu được min P Đẳng thức xảy ra khi và chỉ khi a=b=c=1. 2 Cách 4 Bài toán được giải quyết hoàn toàn chỉ bằng AM−GM. Thật vậy, sử dụng bất đẳng thức AM−GM ta có a3 a 3 b 2 1 3 a 2 • b2 1 b 2 1 2 2 2 b3 b 3 c 2 1 3 b 2 • c2 1 c 2 1 2 2 2 c3 c 3 a 2 1 3 c 2 • a2 1 a 2 1 2 2 2 Cộng vế theo vế các bất đẳng thức trên, ta được 3 3 3 a b c 12 2 2 3 2 2 2 2 a b c 3 a b c Hay tương đương với b2 1 c 2 1 a 2 1 2 2 2 a3 b 3 c 3 3 2 b2 1 c 2 1 a 2 1 2 Bài 88: Cho a,b,c≥0 thỏa mãn a2 b 2 c 2 3. Chứng minh rằng a2 b 2 c 2 3 abc 2( a b c ).
  51. a3 a b 3 b c 3 c Lời giải 2. Sử dụng bất đẳng thức AM-GM, ta có a2 ,,. b 2 c 2 2 2 2 a3 a b 3 b c 3 c Từ đó, bài toán được đưa về chứng minh 3abc 2( a b c ), 2 2 2 tương đương a3 b 3 c 3 6 abc 3( a b c ). do 3 a2 b 2 c 2 nên ta chỉ cần chứng minh được abc3 3 3 6 abc ( a 2 b 2 cabc 2 )( ). Bất đẳng thức này tương đương với bất đẳng thức hiển nhiên đúng ba( b ) bcb ( c ) cac ( a ) 6 abc . Và như thế, phép chứng minh của ta đã được hoàn tất. ■ Bài 89: Cho 3 số thực a,b,c thỏa mãn a2 b 2 c 2 3. Tìm giá trị lớn nhất của: A a b c abc Ta có thể giải như sau: Áp dụng bất đẳng thức Cauchy-Schwarz ta có: 2 2 2 2 2 abcabc ( abc )(1 ab ) ( ab ) c 1(1 ab ) abc2 2 2 2 ab 2 2 abab 2 2 3 2 ab 2 2 abab 2 2 a2 b 2 a 2 b 2 c 2 3 3 Đặt: t=ab với:|ab | | t | 2 2 2 2 15 3 Ta xét hàm số: f( t ) (3 2 t )(2 2 t t2 ) Khảo sát hàm số trên ta được: f() t dấu ′′=′′ xảy ra khi: t Suy ra: 2 2 15 15 3 a b c abc VậyMaxA= đẳng thức xảy ra khi: a b ; c 0 và các hoán vị. 2 2 2 Bài 90: Cho a,b,c là các số dương thỏa mãn a + b + c = 3. Chứng minh rằng 18 2a 4 b 33 c2 9( ab bc ca ) Lời giải: Trước tiên ta đưa bất đẳng thức về dạng:18 2a 4 b 33 c2 9( ab bc ca ) Đứng trước tình hình này ta sẽ nghĩ làm thế nào để đưa được bất đẳng thức trên về dạng đối xứng, như thế có thể sẽ dễ dàng hơn trong việc giải quyết bài toán này!Với suy nghĩ như vậy ta làm như sau: Ta thấy đẳng thức xảy ra khi a=b=c=1 vậy áp dụng bất đẳng thức AM-GM ta có: 2a 2 4 a , 1 33c2 1 3 c 2 3 c 2 3 c 2 4 c Từ đây ta chỉ cần chứng minh:15 4a 4 b 4 c 9( ab bc ca ) Ta đã đưa được bất đẳng thức về dạng đối xứng, đến đây có lẽ bài toán đã dễ dàng hơn rất nhiều rồi, với một chút kinh nghiệm, để chứng minh bất dẳng thức trên ta chia nhỏ ra và chứng minh hai bất đẳng thức sau: ab bc ca 3, a b c ab bc ca
  52. Ta có(a b c )2 3( ab bc ca ) kết hợp với a+b+c=3 ta có ngay ab+bc+ca≤3 như thế bất đẳng thức đầu tiên đã được chứng minh. Bây giờ ta tìm cách chứng minh bất đẳng thức thứ hai! Ta thấy rằng nếu cứ để biểu thức ab+bc+ca thì việc chứng minh khá là khó khăn, bởi vậy ta tìm cách khử nó đi! 9 a2 b 2 c 2 Nhớ tới giả thiết a+b+c=3 ta có thể rút ab bc ca .Thay vào bất đẳng thức hai thì ta phải chứng 2 minh: a2 b 2 c 2 2 a 2 b 2 c 9 Đến đây các bạn có thể thấy ngay ta phải làm tiếp thế nào! Sử dụng AM- GM ta có: a2 2 a a 2 a a 3 a Tương tự như vậy: b2 2 b 3 b , c 2 2 c 3 c Cộng ba bất đẳng thức này lại với chú ý a+b+c=3 ta sẽ được ngay bất đẳng thức trên! Từ hai bất đẳng thức trên ta có:154445( a b c abbcca )4( abbcca )9( abbcca ) Như vậy bất đẳng thức đã cho được chứng minh!Đẳng thức xảy ra khi và chỉ khi a=b=c=1. ■ Bài 91: Cho các số thực dương a,b,c thỏa mãn: 9 a4 b 4 c 4 25 a 2 b 2 c 2 48 0 . Tìm giá trị nhỏ nhất a2 b 2 c 2 (GTNN) của biểu thức: P b 2 c c 2 a a 2 b Bình thường khi tiếp cận 1 bài toán, ta khai thác triệt để giả thiết của bài toán trước, và từ giả thiết sau khi đã khai thác sẽ định hướng cho ta sẽ phải làm gì. Ở bài toán này, với điều kiện giả thiết 9 a4 b 4 c 4 25 a 2 b 2 c 2 48 0 và biểu thức P là đối xứng với các biến, nên ta dự đoán đẳng thức xảy ra khi a=b=c=1. Từ giả thiết 9 a4 b 4 c 4 25 a 2 b 2 c 2 48 0 với bậc của các biến là bậc 4 và bậc 2, nên ta có hai hướng khai khác 1. Đưa các biến về cùng bậc 4. Thật vậy, dựa vào dự đoán dấu bằng và theo bất đẳng thức AM−GM ta có 25 0 9 abc444 25 abc 222 48 9 abc 444 abc 444 3 48 tức là ta có a4 b 4 c 4 3. 2 2. Đưa các biến về cùng bậc 2. Thật vậy, dựa vào dự đoán dấu bằng và theo bất đẳng thức AM−GM ta có 0(9 a4 9)(9 a 4 9)(9 a 4 9)25 abc 222 2118 abc 222 25 abc 222 21 tức là ta có a2 b 2 c 2 3. Như vậy, giả thiết của ta đã khai thác xong và hiện tại ta chẳng biết làm gì với giả thiết đó, và trong 2 điều mà ta khai thác được, ta nên dùng cái nào? Bây giờ ta xét biểu thức P. Nhờ dự đoán đẳng thức xảy ra tại a=b=c=1 và sử dụng giả thiết, nên ta sẽ có ý đồ là chứng minh bất đẳng thức đồng bậc để sử dụng giả thiết. 3 4 4 4 a2 b 2 c 2 4 3 a b c Tức là ta sẽ chứng minh P (1) hoặc là b 2 c c 2 a a 2 b 3 a2 b 2 c 23( a 2 b 2 c 2 ) P (2) Nhận thấy bất đẳng thức (1) mạnh hơn bất đẳng thức (2) vì b 2 c c 2 a a 2 b 3 2 a2 b 2 c 2 a4 b 4 c 4 a 2 b 2 c 2 , nhưng liệu ta có nhất thiết phải dùng một đánh giá mạnh hơn (2) 3 như thế hay không? Mà chắc gì (1) đã đúng
  53. Để xuất hiện cái a2 b 2 c 2 hì theo lẽ tự nhiên ta sẽ sử dụng Cauchy−Schwarz để trên tử số xuất hiện a2 b 2 c 2 trước đã, mẫu thế nào tính sau . Sử dụng bất đẳng thức Cauchy−Schwarz ta có: a2 b 2 c 2 P b 2 c c 2 a a 2 b a4 b 4 c 4 a2( b 2 c ) b 2 ( c 2 a ) c 2 ( a 2 b ) 2 a2 b 2 c 2 abbcca2 2 2 2 acbacb 2 2 2 Ô la la, trên tử xuất hiện a2 b 2 c 2 rồi . Còn cái mẫu thì sao nhỉ? Lại một lần nữa ngài Cauchy−Schwarz cho mẫu số của chúng ta xuất hiện a2 b 2 c 2 Thật vậy, tiếp tục sử dụng bất đẳng thức Cauchy−Schwarz ta có: 2 a2 b 2 c 2 a 2 b 2 c 2 a 2 b 2 c 2 abbcca2 22 abcabbcca 222222222 abc 222 . 3 3 2 a2 b 2 c 2 a 2 b 2 c 2 tương tự, ta cũng có 2 a2 c b 2 a c 2 b Do đó 3 2 2 a2 b 2 c 2 a 2 b 2 c 2 abbcca2 2 2 2 acbacb 2 2 2 3 a2 b 2 c 2 a 2 b 2 c 2 Như vậy, cái đơn giản đã đưa ta đi đến 3 a2 b 2 c 2 3 thành công, thật may mắn . a2 b 2 c 23( a 2 b 2 c 2 ) Sử dụng giả thiết mà ta khai thác được là a2 b 2 c 2 3 ta có P 1 b 2 c c 2 a a 2 b 3 Đẳng thức xảy ra khi và chỉ khi a=b=c=1. Vậy minP=1. Bài toán được giải quyết hoàn toàn. ab bc ca Bài 92: Cho a,b,c dương và a2 b 2 c 2 3.Chứng minh: 3 c a b abbc bcca caab Để ý rằng .;.;. b2 c 2 a 2 mà giả thiết của ta là a2 b 2 c 2 3 nên một lẽ tự nhiên, ta sẽ cộng c a a b b c ab bc bc ca ca ab vế với vế các đẳng thức trên để sử dụng được giả thiết. . . . a2 b 2 c 2 3 Bây giờ đặt: c a a b b c ab bc ca x ,, y z thì ta có xy+yz+zx=3 và bất đẳng thức cần chứng minh tương đương với x y z 3 Bài c a b toán đã trở nên sáng tỏ bởi ta luôn có:(x y z )2 3( xy yz zx ) 3.3 Tức là, x y z 3 Phép chứng minh hoàn tất. Đẳng thức xảy ra khi và chỉ khi x=y=z hay a=b=c=1.
  54. Bài 93: Cho các số thực a,b,c không âm và có tổng a + b + c = 3. Tìm giá trị lớn nhất của biểu thức : P a2 a 4 b 2 b 4 c 2 c 4 2x 6 Gợi ý. Sử dụng đánh giá sau: x2 x 4 BĐT ( hay Cực trị) đa phần sẽ đạt được tại "tâm" hay tại "biên". 3 Do đó, sau một hồi lâu "thử nghiệm" thì có tìm được "Đáp số" như nguoivn. Thế là, ta mong muốn có đánh giá x2 x 4 ax b . Mà "dấu bằng" xảy ra tại x=3 và x=0. Thế là, ta có ngay, a=23 và b=2. (x y )(1 xy ) Bài 94: Tìm Max và Min của biểu thức: (1 x2 )(1 y 2 ) a b c b c a Bài 95: Cho a,b,c làđộ dài 3 cạnh của 1 tam giác. Chứng minh rằng 3 2 3 b c a a b c a(1 bcca )(1 b caab )(1 c abbc ) 27 Bài 96: Cho a,b,c>0 và abc=1 Chứng minh rằng: b3 c 3 a 3 a 3 b 3 c 3 Bất đẳng thức cần chứng minh trương đương với a(1 bc ca ) b (1 ca ab ) c (1 ab bc ) (a3 b 3 c 3 ) 27 3 3 3 b c a Sử dụng bất đẳng thức AM-GM, ta có a3 b 3 c 3 3 abc 1 nên ta chỉ cần chứng minh a(1 bcca ) b (1 caab ) c (1 abbc ) 9 b3 c 3 a 3 Cũng theo bất đẳng thức AM-GM thì a(1 bc ca ) 3 a . 3 abc2 b3 b 3 b(1 ca ab ) 3 b . 3 bca2 c3 c 3 c(1 ab bc ) 3 c .3 cab2 a3 c 3 a 3 abc2 b 3 bca 2 c 3 cab 2 Vậy nên ta chỉ còn chứng minh 3 Thế nhưng điều này là hiển nhiên đúng theo bất b3 c 3 a 3 đẳng thức AM-GM cho ba số với giả thiết abc=1. Bài toán được chứng minh xong .□ Bài 98: Chứng minh rằng với mọi số thực không âm a,b,c, ta luôn có 2(a2 b 2 )( b 2 c 2 )( c 2 a 2 ) ( ab 2 bc 2 ca 2 abc ) 2 Ta có thể đi theo hướng khác như sau: Vì ở VT có xuất hiện các đại lượng như: (a2 b 2 ); ( b 2 c 2 ); ( c 2 a 2 ) nên ta sẽ cố gắng áp dụng bất đẳng thức Cauchy-Schwarz vào VP sao cho xuất hiện một trong các đại lượng trên, như vậy việc chứng minh sẽ nhẹ nhàng
  55. hơn nhiều. Từ ý tưởng đó, ta sẽ áp dụng bất đẳng thức Cauchy-Schwarz, ta có: 2 2 2 2 2 2 2 2 2 2 2 ()()()()()()ab bc ca abc a b ca c bc ab a c b ca bc ab Do vậy ta chỉ cần chứng minh được: 2(a2 bb 2 )( 2 c 2 ) ( b 2 ca ) 2 ( bcab ) 2 Khai triển ra ta thu được: a2 b 2 b 2 c 2 c 2 a 2 b 4 4 ab 2 c Áp dụng AM-GM ta thu được: ab2222224 bc ca b 44 abc 484 4 abc 2 hứng minh hoàn tất. Đẳng thức xảy ra khi: a=b=c. Từ đó ta có chú ý sau: Khi sử dụng bất đẳng thức Cauchy-Schwarz ta sẽ cố gắng tạo ra các đại lượng ở hai vế giống nhau để giảm bớt đi, như vậy việc chứng minh sẽ nhẹ nhàng hơn rất là nhiều, mà ví dụ trên là một ví dụ điển hình. 2 2 2 Bài 99: Cho các số thực a,b,c thỏa mãn 6a 3b 2c abc 24 Chứng minh rằng: (a 1)( b 4)( c 9) 144 Bất đẳng thức cần chứng minh tương đương với: 2(a2 1)( b 2 4)( c 2 9) (6 a 3 b 2 c abc ) 2 Áp dụng bất đẳng thưc Cauchy-Schwarz vào VP ta được: 22 2 2 2 (6abcabc 3 2 )  abc (6 )(3 bc 2) ( a 1)(6 bc )(3 bc 2) Ta cần chứng minh: 2(b2 4)( c 2 9) (6 bc ) 2 (3 b 2 c ) 2 Khai triển tương đương ta có:(bc 6)2 (3 b 2 c ) 2 0 Hiển nhiên đúng. Điều phải chứng minh. (a b c )3 4( a b c ) 4 5 Bài 100: Chứng minh bất đẳng thức sau đúng với mọi a,b,c dương, c . ab() a c a b a c Do biểu thức đối xứng với 2 biến a,c và để ý rằng bất đẳng thức trên đồng bậc 0 nên ta sẽ đưa bất đẳng thức trên về 2 x,y biến để giải quyết. Vậy 2 biến đó là 2 biến nào? Ta để ý rằng vì đối xứng giữa 2 biến a,c nên ta nghĩ tới việc dấu ′′=′′ xảy ra khi: a=c, do đó khi ta đưa về biến x,y thì cũng mong sao 2 biến đó có dấu ′′=′′ xảy ra khi: x=y. Nên khi chuyển vế 2 biến thì vai trò của a,c vẫn phải được đảm bảo bình đẳng như cũ, có như vậy bài toán mới dễ giải a c quyết. Từ những phân tích đó gợi ý cho ta đưa về 2 biến: x ;. y Thật vậy ta đưa bất đẳng thức ban đầu về b b 3 a c a c c 1 4 1 5 b b b b 4c dạng: b a a c ab a c b b b b b b a c Đặt: x ;. y Ta thu được bất đẳng thức khá gọn và đẹp như sau: b b 3 x y 1 4(x y 1) 5 y 4y ( xy 1)4(3 xyxy )( 1)4( xyxyxy )4 x() x y x x y Ta lại nhận thấy rằng bất đẳng thức trên đậm màu Viet khi liên tục chứa các biểu thức: x+y;xy, nên tư tưởng đưa về hàm bậc nhất nó hiện hữu ngay trước mắt, giống như miếng mồi ngon mà không ăn ngay để lâu sẽ bị người khác ăn
  56. mất! Ta sẽ chuyển bất đẳng thức về dạng Viet thông qua việc đặt ẩn phụ chứ nếu để như trên và biến đổi thì khá cồng S2 kềnh.Đặt S=x+y;P=xy. Để ý rằng với việc đặt như trên ta luôn có: P . Bất đẳng thức trên trở thành: 4 (1)4(1)4S 3 SS SPP 5 (54) SPSSS 3 2 10 Dễ thấy rằng bất đẳng thức trên chỉ là một hàm bậc nhất với biến P, nên ta sẽ đánh gia thông qua hàm số bậc nhất này. Các bạn lưu ý rằng hàm bậc nhất là hàm dễ đánh giá nhất, vì nó luôn luôn đơn điệu nên ta chỉ cần quan tâm tới 2 biên của nó là đủ.Ta xét hàm số: f( P ) (5 4 S ) P S3 S 2 S 1 S 2 Với: 0 P Ta sẽ xét hai biên của nó.Ta có: 4 f(0) S3 S 2 S 1 ( S 1) 2 ( S 1) 0 2 2 2 SSS 3 2 1 2 f (5 4 S ) S S S 1 S 1 S 2 0 4 4 4 4 Do cả hai biên của hàm số f(P) đều không âm nên ta có điều phải chứng minh. Dấu ′′=′′ xảy ra khi: S=2⇔x=y=1⇔a=b=c. 2 2 3 a c 4 a d 9 Bài 101: Cho a,b,c,d ∈ [3;4].Tìm GTLN, GTNN của: T 4 d a 3 b a 16 Mình sẽ gợi ý giải phần tìm maxT, phần tìm minT bạn làm tương tự nhé: Từ giả thiết ta có a c a 4 4 d 4 d 7 0 1 1 a d a d a d3 d 3 d Và a d a d d 3 d 3 d 3 0 1 1 . a b a 3 a 3 3 3 6 Do đó 2 2 3 7 4 d 3 9 T 4 d 3 3 6 16 và đẳng thức xảy ra khi a=3,b=3,c=4. Bây giờ, ta chỉ việc khảo sát hàm 2 2 3 7 4 d 3 9 f() d rên [3,4] nữa là xong. Giá trị lớn nhất của f(d) trên đoạn này sẽ đạt được tại 4 d 3 3 6 16 một d0 nào đó thuộc [3,4] và giá trị lớn nhất này cũng sẽ chính là giá trị lớn nhất của T vì ta có đẳng thức xảy khi a=b=3,c=4 và d=d0. Bạn tự khảo sát hàm và tính giá trị này nhé. 1 1 1 1 Bài 102: Cho a,b,c>0 thỏa mãn abc=1. Chứng minh rằng : 1 (1 a )2 (1 b ) 2 (1 c ) 2 ab bc ca 1 1 1 c Sau khi đã đánh giá được (1 a )2 (1 b ) 2 1 c 1 1 1 1 c Lại để ý rằng ab bc ca 1 1 1 c 2 1c( a b ) 1 c (1 ab ) 1 c 2 ( 1) c c c Tới đây ta đã thu được bất đẳng thức một biến c. Phần còn lại rất đơn giản, bạn tự hoàn tất nhé a bc b ca c ab Bài 103: Cho a,b,c>0 thoả mãn a + b + c = 1. Tìm GTLN của: T a bc b ca c ab
  57. Ta có: a bc b ca c ab 3 T 1 1 1 a bc b ca c ab 2bc 2 ca 2 ab a bc b ca c ab b2 c 2 c 2 a 2 a 2 b 2 2( ) Q abc bc2 2 abc ca 2 2 abc ab 2 2 Áp dụng BĐT Cauchy-Schwarz: 2(ab bc ca )2 Q 3abc a2 b 2 b 2 c 2 c 2 a 2 Chúng ta đi chứng minh: 2(ab bc ca )2 3 3abc a2 b 2 b 2 c 2 c 2 a 2 2 Hay ,sau khi nhân chéo và rút gọn : ab2 2 bc 2 2 ca 2 2 8 abcabc ( ) 9 abc (1) Bây giờ áp dụng giả thiết a+b+c=1, ta được: (1) ab2 2 bc 2 2 ca 2 2 8 abcabc ( ) 9 abcabc ( ) ab2 2 bc 2 2 ca 2 2 ( abcabc ) (2) 3 (2) đúng theo AM−GM do đó, suy ra: Q 2 3 3 3 1 Hay: 3 TT Vậy: MaxT đạt được khi: a b c 2 2 2 3 a b c Bài 104: Cho các số thực dương a,b,c thỏa mãn abc=1. Chứng minh rằng 1 a b 1 b c 1 c a 1 Sau một hồi suy nghĩ khá lâu, mình nhận thấy rằng bài này chắc chắn là dùng Cauchy-Schwarz rồi tuy nhiên nếu để "nguyên đai, nguyên kiện" thế kia thì chắc là không thu được kết quả gì. Như thế chắc chắn phải đổi biến rồi, với giả thiết abc=1 ta nhớ ngay tới phép đổi biến: y z x a ,, b c x y z Thay a,b,c như trên vào thì ta phải chứng minh: y2 z 2 x 2 P 1 y2 xz zy z 2 xy yz x 2 xy xz Đến đây thì "thơm" quá rồi, áp dụng trực tiếp Cauchy-Schwarz ta có: ()x y z 2 P 1 x2 y 2 z 2 2 xy 2 yz 2 zx Vậy là bài toán chứng minh xong! Đẳng thức xảy ra khi và chỉ khi a=b=c=1 Cách 2 Dạng này khá quen thuộc nhỉ! Theo lời gợi ý của anh Cẩn thì chúng ta có lời giải như sau: x y z Đặt a ,; c b y z x Khi đó bất đẳng thức cần chứng minh tương đương với: xz xy yz 1 xz y2 yz xy z 2 xz yz x 2 xy Áp dụng bất đẳng thức Cauchy Schwarz ta có:
  58. xz xy yz xz y2 yz xy z 2 xz yz x 2 xy ()()()()xz2 xy 2 yz 2 xy yz zx 2 1 ()xz222222 xy z xyz () xy xyz x yz () yz 222 x xy xy z ()()()2( xy 222 yz xz xyz x y z ) Điều phải chứng minh. Đẳng thức xảy ra khi: x=y=z⇔a=b=c. Bài 105: Chứng minh rằng với mọi số thực a,b,c không âm ta luôn có: (abc )4 8( a 2 b 2 cabbcca 2 )( ) Ta có: 8(a2 b 2 cabbcca 2 )( )4( a 2 b 2 c 2 )(222)( ab bc ca a 2 b 2 c 2 222) ab bc ca 2 8(a2 b 2 c 2 )( abbcca ) ( abc ) 4 Dấu bằng xảy ra khi: a2 b 2 c 2 2 ab 2 bc 2 ca Bài 106: Cho a,b,c là các số thực thoản mãn a,b,c>1 và a + b + c = abc. Chứng minh rằng: (a2 1)( b 2 1)( c 2 1) 8 a b c Bài 107: Cho a,b,c>0 thỏa mãn: a + b + c = 3. Chứng minh rằng: 1 a3 b 2 c b 3 c 2 a c 3 a 2 b Áp dụng bất đẳng thức Cauchy - Schwarz, ta có 3 2 1 2 (a b c ) 1 c ( a b c ) , a a1 a ca từ đó ta suy ra . Do đó để kết thúc chứng minh, ta cần chỉ ra rằng a3 b 2 c() a b c 2 3 a b c ab bc ca 1, tuy nhiên đây lại là một đánh giá dễ dàng chứng minh nếu ta để ý rằng ab+bc+ca≤3. ()a b c 2 ■ 3 a b c 8( a bc )( b ca )( c ab ) Bài 108: Cho a,b,c>0. Chứng minh rằng: 1 3 (a b )( b c )( c a ) Gợi ý. - Bước 1. Sử dụng kĩ thuật ghép đối xứng, hãy chứng minh 8(abcbcacab )( )( ) ( abbccaa )( )( )( 1)( b 1)( c 1) Cụ thể (abc )( 1) abcbca 2 ( abcbca )( ) Viết 2 BĐT tương tự rồi nhân theo vế ta có điều phải chứng minh. - Bước 2. Sau khi sử dụng đánh giá trên, đặt a+1=x, b+1=y, c+1=z, ta sẽ thu được 3 x y z xyz 3 1 Bài 109: Cho x≥1.Tìm giá trị nhỏ nhất của biểu thức: y 3 x 2x Bài toán khá đơn giản, áp dụng AM-GM cũng được, nhưng tự nhiên nhất là sử dụng hàm số. 1 1 6x2 1 Xét hàm số: f( x ) 3 x Ta có: f ( x ) 3 0 2x 2x2 2 x 2 Do đó hàm số đồng biến với mọi x≥1. 7 Suy ra: f( x ) f (1) 2
  59. Bài 110:Cho x,y,z là các số thực dương thỏa mãn x + y + z = 3. Tìm giá trị lớn nhất của biểu thức ()()()x y2 y z 2 z x 2 P . x2 y 2 z 2 3 Để ý thấy P là một biểu thức đối xứng, mặt khác x,y,z>0 nên dễ dàng Gia Cát Dự MaxP=2, đạt được khi x=y=z=1. Tức ta cần chỉ ra ()()()x y2 y z 2 z x 2 2 Hay2(x2 y 2 z 2 3) ( x y ) 2 ( y z ) 2 ( z x ) 2 x2 y 2 z 2 3 Tương đương với xy+yz+zx≤3 ()x y z 2 Áp dụng bổ đề quen thuộc xy yz zx 3 ta suy ra ngay đpcm. 3 x y y z z x Bài 111: Cho x,y,z ∈ [1,2]. Tìm giá trị lớn nhất và giá trị nhỏ nhất của biểu thức: P 2 z 2 x 2 y Mình tìm GTNN trước: x y x y Với điều kiện trên ta có: 2≤x+y. Suy ra: Tương tự như trên ta có: 2 z x y z xyyzzx xy yz zx P 2 Suy ra: MinP=2, Dấu ′′=′′ xảy ra khi: x=y=z=1. 2 z 2 x 2 yxyzyzxzxy Bài 112: Cho các số thực không âm a,b,c thỏa mãn a2 b 2 c 2 1. Tìm giá trị lớn nhất của biểu thức P a2 b b 2 c c 2 a. Cách 1: ()x y z 2 Sử dụng bất đẳng thức Cauchy−Schwarz và đánh giá quen thuộc xy yz zx ta có: 3 (a2 b 2 c 2 ) 2 1 P2 ( )( ababcbcac 2222222222 abbccaabc )( ) .( abc 222 ) 3 3 1 1 1 Do đó P Vậy maxP khi a b c 3 3 3 a3 b 3 c 3 a b c Bài 113: Cho a,b,c dương. Chứng minh a2 abb 2 b 2 bcc 2 c 2 aca 2 3 Ta có: a3 b 3 b 3 c 3 c 3 a 3 0 a2 ab b 2 b 2 bc c 2 c 2 ac a 2 a3 b 3 c 3 b 3 c 3 a 3 a2 ab b 22 b bc c 22 c ac a 22 a ab b 22 b bc c 22 c ac a 2 Vậy bất đẳng thức cần chứng minh tương đương với bất đẳng thức sau : 1 a3 b 3 b 3 c 3 c 3 a 3 a b c 2a2 ab b 2 b 2 bc c 2 c 2 ac a 2 3 a2 ab b 2 b 2 bc c 2 c 2 ac a 2 2( a b c ) ()()()a b b c c a a2 ab b 2 b 2 bc c 2 c 2 ca a 2 3 Hiển nhiên ta có:
  60. aabb2 21 bbcc 2 2 1 aacc 2 2 1 ;; aabb2 23 bcbc 2 2 3 aacc 2 2 3 Từ đó ta suy ra bất đẳng thức trên đúng.Dấu bằng xảy ra khi a=b=c Vậy ta có điều phải chứng minh. Cách 2 a3 abba 2 2 abab()() ab a a a a2 ab b 2 a 2 ab b 2 3 ab 3 Tương tự cho 2 vế còn lại rồi cộng lại, từ đó ta có điều phải chứng minh. Bài 114: Cho a,b,c là các số thực thỏa mãn a2 b 2 c 2 ab bc ca 1 . Tìm giá trị nhỏ nhất, giá trị lớn nhất của biểu thức P a2 b 2 4 c 2 . 7 33 7 33 Gợi ý. Sử dụng phương pháp tam thức bậc hai Ta sẽ thu được kết quả: maxPP ,min 2 2 1 Bài 115: Cho các số thực a,b,c đôi một khác nhau thỏa mãnb2 bc ca ab . Tìm giá trị nhỏ nhất của biểu 2 2 2 2 a b c thức: P 4. 9. a b b c c a Ta có: 1 1 b2 bc ca ab b 2 bc ab ac ab 2 2 a b 2(a b )( b c ) ab . 2 a b b c a b c a b Khi đó áp dụng BĐT AM - GM ta có: P ( )4(2 )9( 2 )4 2 . 8 a b b c c a a b b c Dấu = xảy ra khi (a,b,c)=(2,1,0). Bài 116: Cho các số thực không âm a,b thỏa mãn a2 b 2 1. Tìm giá trị lớn nhất của biểu thức 2 2 2 2 a b P a (). a b và Q a . 2 Ta có a2 ( a b ) 2 2 a 2 2 ab b 2 P a2 b 2 a 2 b 2 Nếu b=0 thì P=2. t2 2 t 1 a Nếu b≠0 thì P , t Hay là t2 1 b (P 2) t2 2 t P 1 0 Phương trình theo ẩn t luôn có nghiệm, do đó
  61. 1 (PP 2)( 1) 0 PP2 3 1 0 3 5 P 2 5 5 5 5 3 5 Đẳng thức xảy ra khi a , b vậy maxP 10 10 2 Cách 2 Thật vậy, với giả thiết a2 b 2 1 ta có thể đặt: a cos , b sin với α∈[0,2π] Lúc đó ta có: 2sin 2 cos2 3 P 2 sin 2 2cos2 3 Q 4 Phần việc còn lại là đi tìm GTLN của P,Q. Đây là một việc khá đơn giản, mình xin để các bạn tự hoàn thiện! Bài 117: Cho a,b,c≥0 thỏa mãn: a+b+c=1.Tìm giá trị lớn nhất và giá trị nhỏ nhất của biểu thức sau: Fabc(,,)()()() abc 3 bca 3 cab 3 Bài 118: Cho a,b,c≥0 thỏa mãn c2 a 2 ab b 2 . Chứng minh rằng a3 b 3 3 abc 5 c 3 Ta thấy rằng: Khi c=0 thì: a2 ab b 2 0 ( a b ) 2 ab 0 a b 0 Bất đẳng thức trở thành đẳng thức. Do vậy ta chỉ cần xét khi: c>0. Khi đó ta có nhận xét rằng với bất đẳng thức trên ta thấy nó vừa là bất đẳng thức ở dạng đồng bậc và đối xứng với 2 biến: a,b nên ta có thể chuyển về 2 biến để giải quyết. Ta sẽ giải quyết bài toán như sau: 2 2 a ab b Đưa điều kiện về dạng: 2 1 c c c 3 3 a b ab Tương tự ta đưa bất đẳng thức về dạng: 32 5 c c c a b Đăt: x ; y Bài toán ban đầu được đưa về bài toán sau: c c Cho 2 số thực không âm x,y thỏa mãn: x2 xy y 2 1 Chứng minh rằng: x3 y 3 3 xy 5 Ta đã có: x+y>0, nên nhân (x+y) vào điều kiện ta có: xyxxyy )(2 2 ) xy xy 3 3 xy Do đó ta có: ()x y 2 x3 y 3 3 xy 5 x y 3 xy 5 Mà theo điều kiện ta có:1 x2 xy y 2 ( x y ) 2 4 x y 2 4 3(x y )2 Từ đó dẫn tới: x y 3 xy x y 2 3 5 4
  62. 1 Bài 119: Cho các số thực a,b,c khác nhau đôi một thỏa mãn b2 bc ca ab . 2 2 2 2 a b c Tìm giá trị nhỏ nhất của biểu thức: P 4. 9. a b b c c a Bài 120: Cho các số thực không âm a,b,c thỏa mãn a+b+c=2. Tìm giá trị lớn nhất của biểu thức: P ( ab3 3 bc 3 3 ca 3 3 )( ab bc ca ) Bài 123: Cho a,b,c≥0 thỏa mãn a+b+c=2. Chứng minh rằng: (a2 bc )( b 2 ca )( c 2 ab ) 1